Annotation of db/baza/mgp11718.txt, revision 1.3

1.1       rubashki    1: Чемпионат:
                      2: Первая лига Москвы сезона 2017/18 гг.
                      3: 
                      4: Дата:
                      5: 23-Dec-2017 - 19-May-2018
                      6: 
                      7: Тур:
                      8: 1.1. "Разведка боём"
                      9: 
                     10: Дата:
                     11: 23-Dec-2017
                     12: 
                     13: Редактор:
                     14: Мария Подрядчикова (Волгоград) и Вадим Яковлев (Москва)
                     15: 
                     16: Инфо:
                     17: Редакторы благодарят за тестирование и ценные замечания Дарью Харченко
                     18: (Волгоград), Георгия Матюшина, Вадима Туха, Богдана Чадромцева (все -
                     19: Санкт-Петербург), Гая Кузнецова, Александру Семёнову, Алексея Сироткина,
                     20: Алексея Тырышкина (все - Москва). Особая благодарность выражается Ивану
                     21: Митрофанову (Щёлково) и Максиму Мерзлякову (Воронеж).
                     22: 
                     23: Вопрос 1:
                     24: Рассказывая о задержании девушек на таможне, Олег Кирьянов отмечает, что
                     25: те не огорчились, поскольку смогли убедиться в успешности своих АЛЬФ.
                     26: Назовите АЛЬФУ двумя словами, начинающимися на соседние буквы.
                     27: 
                     28: Ответ:
                     29: Пластическая операция.
                     30: 
                     31: Комментарий:
                     32: Девушек задержали из-за того, что после пластических операций они
                     33: выглядели симпатичнее, чем в паспорте.
                     34: 
                     35: Источник:
                     36: О.В. Кирьянов. Корея и корейцы. О чем молчат путеводители.
                     37: http://flibusta.is/b/392530/read
                     38: 
                     39: Автор:
                     40: Мария Подрядчикова (Волгоград)
                     41: 
                     42: Вопрос 2:
                     43: Иэн Макьюэн сравнивает с НЕЙ изуродованное после падения с большой
                     44: высоты лицо мертвеца. Какую из НИХ называли "страшным обвинением
                     45: фашизму"?
                     46: 
                     47: Ответ:
                     48: "ГернИку".
                     49: 
                     50: Комментарий:
                     51: ОНА - картина Пабло Пикассо. Взглянув на изуродованное лицо, герой
                     52: Макьюэна осознал радикальное, в духе Пикассо, нарушение законов
                     53: перспективы. Картина "ГернИка" была написана после бомбардировки
                     54: одноименного города в ходе гражданской войны в Испании.
                     55: 
                     56: Источник:
                     57:    1. И. Макьюэн. Невыносимая любовь. http://flibusta.is/b/107171/read
                     58:    2. https://ru.wikipedia.org/wiki/Герника_(картина)
                     59: 
                     60: Автор:
                     61: Мария Подрядчикова (Волгоград)
                     62: 
                     63: Вопрос 3:
                     64: (pic: 20170773.jpg)
                     65:    По розданной вам фотографии неспециалисту сложно определить,
                     66: пользуется ли изображенный на ней человек своей продукцией. Назовите
                     67: этого человека.
                     68: 
                     69: Ответ:
                     70: [Кристиан] Лубутен.
                     71: 
                     72: Комментарий:
                     73: Красный цвет является "визитной карточкой" модельера, что прослеживается
                     74: даже в окружающем его интерьере. Тем не менее, зрителю не видны подошвы
                     75: его ботинок, поэтому точно определить, от Лубутена ли они, сможет разве
                     76: что эксперт в области моды. Лубутен производит как женские, так и
                     77: мужские туфли, а также одежду и аксессуары.
                     78: 
                     79: Источник:
                     80: https://www.tatler.ru/geroi/tatler-v-gostyah-u-kristiana-lubutena-v-parizhe
                     81: 
                     82: Автор:
                     83: Вадим Яковлев (Москва)
                     84: 
                     85: Вопрос 4:
                     86: Валерий Чкалов желал сплотить экипаж своей арктической экспедиции и
                     87: поначалу даже собирался взять на борт ЕГО, однако потом предпочел ЕМУ
                     88: большой запас коньяка. Назовите ЕГО двумя словами.
                     89: 
                     90: Ответ:
                     91: Пуд соли.
                     92: 
                     93: Комментарий:
                     94: Чкалов в буквальном смысле загрузил в самолет пуд соли в качестве
                     95: символа дружбы и единства. Несмотря на уговоры товарищей, авиатор никак
                     96: не хотел избавляться от лишнего груза. Тогда штурман вытащил из самолета
                     97: аварийный запас коньяка примерно той же массы, и Чкалов убрал соль.
                     98: 
                     99: Источник:
                    100: Документальный фильм "Люди, сделавшие Землю круглой", 2-я серия.
                    101: https://www.youtube.com/watch?v=c4T-ChiBWNg&t=33m21s
                    102: 
                    103: Автор:
                    104: Иван Митрофанов (Щёлково)
                    105: 
                    106: Вопрос 5:
                    107: В восемнадцатом веке европейская часть России была густо населена, а вот
                    108: за Уралом до самого Дальнего Востока тянулась лишь узкая полоса обжИтой
                    109: земли. Вадим Нестеров сравнил Россию с НИМ. В честь НЕГО получил
                    110: название гидросамолет. Назовите ЕГО одним словом.
                    111: 
                    112: Ответ:
                    113: Головастик.
                    114: 
                    115: Комментарий:
                    116: Карта России в начале восемнадцатого века напоминала головастика -
                    117: относительно равномерно заселенная европейская часть и узкая полоса
                    118: Земли от Урала и до Дальнего Востока. Гидросамолет назвали в честь
                    119: представителя амфибий.
                    120: 
                    121: Источник:
                    122:    1. В. Нестеров. Люди, принесшие холод. Книга первая: Лес и Степь.
                    123: http://flibusta.is/b/362038/read
                    124:    2. https://en.wikipedia.org/wiki/Grumman_G-65_Tadpole
                    125: 
                    126: Автор:
                    127: Мария Подрядчикова (Волгоград)
                    128: 
                    129: Вопрос 6:
                    130: Организация "НатИв" появилась после Второй мировой войны, а крупных
                    131: уступок от стран социалистического блока добилась лишь в перестроечные
                    132: времена. Некоторые вдохновленные "Нативом" митинги проходили под
                    133: лозунгом, неоднократно повторяющимся в американском произведении.
                    134: Напишите этот лозунг.
                    135: 
                    136: Ответ:
                    137: Let my people go.
                    138: 
                    139: Зачет:
                    140: Отпусти народ мой; отпусти мой народ.
                    141: 
                    142: Комментарий:
                    143: "Натив" - это израильская служба, занимающаяся репатриацией евреев.
                    144: Подобно пророку Моисею, "Натив" выводил евреев из стран соцлагеря около
                    145: сорока лет. "Отпусти народ мой" - это фраза из Библии, повторяющаяся в
                    146: песне "Go down Moses" [гОу дАун мОзес]. Кстати, слово "натив"
                    147: переводится с иврита как "дорога", но если вы услышали созвучие со
                    148: словом "нация", то это тоже хорошо.
                    149: 
                    150: Источник:
                    151:    1. Журнал "Ерушалаим-Москва", сентябрь 2017 г.
                    152:    2. https://ru.wikipedia.org/wiki/Go_Down_Moses
                    153: 
                    154: Автор:
                    155: Вадим Яковлев (Москва)
                    156: 
                    157: Вопрос 7:
                    158: Герой Роберта Штильмарка, чудом спасшийся от английских каперов после
                    159: встречи с ИКСОМ, не решился помолиться за соотечественников, которые ему
                    160: помогли. Какие два слова мы заменили ИКСОМ?
                    161: 
                    162: Ответ:
                    163: Летучий голландец.
                    164: 
                    165: Комментарий:
                    166: Голландский капитан так и не смог для себя определить, будет ли молитва
                    167: за неприкаянные души угодна Богу. Англия и Голландия были соперничающими
                    168: морскими державами.
                    169: 
                    170: Источник:
                    171: Р.А. Штильмарк. Наследник из Калькутты. http://flibusta.is/b/472069/read
                    172: 
                    173: Автор:
                    174: Вадим Яковлев (Москва)
                    175: 
                    176: Вопрос 8:
                    177:    <раздатка>
                    178:    #@love
                    179:    </раздатка>
                    180:    Мы раздали вам измененный хэштэг, который был использован во время
                    181: поиска подруги для Джереми. При этом проблемы Джереми объяснялись с
                    182: помощью стрелки. Напишите пять букв, которые мы заменили одним символом.
                    183: 
                    184: Ответ:
                    185: Snail.
                    186: 
                    187: Комментарий:
                    188: Джереми - это улитка с раковиной, закрученной против часовой стрелки.
                    189: Из-за этой мутации Джереми не мог спариваться с обычными улитками, и
                    190: искать ему подругу пришлось при помощи Твиттера. Кстати, раковины всех
                    191: детенышей Джереми были закручены по часовой стрелке. Английское слово
                    192: "snail" [снЭйл] переводится как "улитка", а сам хэштэг звучал как
                    193: snaillove [снЭйл лав] - улиточья любовь. Логика замены обусловлена тем,
                    194: что символ "коммерческое "эт"" в некоторых языках называют улиткой.
                    195: 
                    196: Источник:
                    197:    1. https://nplus1.ru/news/2017/10/14/rip-Jeremy
                    198:    2. https://ru.wikipedia.org/wiki/@
                    199: 
                    200: Автор:
                    201: Вадим Яковлев (Москва)
                    202: 
                    203: Вопрос 9:
                    204: Иронизируя над поражением Хиллари Клинтон, Эрик Трамп отметил, что в
                    205: зале, из которого та планировала прочитать победную речь, ОН. Назовите
                    206: ЕГО двумя словами.
                    207: 
                    208: Ответ:
                    209: Стеклянный потолок.
                    210: 
                    211: Комментарий:
                    212: "Стеклянным потолком" называют негласные барьеры, препятствующие занятию
                    213: женщинами высоких должностей. По совпадению, в зале для собраний, где
                    214: должна была выступить Клинтон, потолок был стеклянным в буквальном
                    215: смысле. Мы же желаем командам пробить потолок и выиграть путевку в
                    216: Высшую лигу.
                    217: 
                    218: Источник:
                    219:    1. https://www.youtube.com/watch?v=yW5-WsR_pGg&t=4m43s
                    220:    2. https://twitter.com/reenajade/status/796108817327157249/
                    221: 
                    222: Автор:
                    223: Вадим Яковлев (Москва)
                    224: 
                    225: Тур:
                    226: 1.2. "МГТУ"
                    227: 
                    228: Дата:
                    229: 23-Dec-2017
                    230: 
                    231: Редактор:
                    232: Дмитрий Смирнов (Москва)
                    233: 
                    234: Инфо:
                    235: Редактор выражает благодарность за помощь в работе над пакетом вопросов
                    236: тестерам: команде "От винта - Братья по фазе" (Харьков), Михаилу Локшину
                    237: (Санкт-Петербург), Алексею Тугареву (Орел), Денису Карпенко (Москва),
                    238: Юрию Чулинину (Новосибирск - Поварово).
                    239: 
                    240: Вопрос 1:
                    241:    <раздатка>
                    242:    Рисия пхуроро о морёстыр,
                    243:    А пхурьятэ нэво уж копано.
                    244:    Нэ пхури уж бутыр закостяпэ:
                    245:    "Дылыно жэ ту, [ПРОПУСК]!
                    246:    Выманглян, дылыно ту, копано!
                    247:    Бут ли сы кофо амэнгэ дрэ лэстэ?
                    248:    Дылыно рисёв ко мачё ту,
                    249:    Пэр прэ чянга, манг ту о кхэр".
                    250:    </раздатка>
                    251:    Перед вами фрагмент известного произведения в переводе на цыганский
                    252: язык. Восстановите десятибуквенное слово в четвертой строке этого
                    253: фрагмента.
                    254: 
                    255: Ответ:
                    256: Простофиля.
                    257: 
                    258: Комментарий:
                    259: Вам был роздан отрывок из "Сказки о рыбаке и рыбке" А.С. Пушкина в
                    260: переводе на цыганский язык. Слово "простофиля" пришло в цыганский язык
                    261: из русского и осталось там в изначальном виде как заимствованное слово.
                    262: 
                    263: Источник:
                    264: http://www.liloro.ru/library/pushkin2.htm
                    265: 
                    266: Автор:
                    267: Алексей Тугарев (Орел)
                    268: 
                    269: Вопрос 2:
                    270: (pic: 20170774.jpg)
                    271:    В 1903 году этому вдумчивому юноше исполнилось 26 лет. Напишите его
                    272: фамилию.
                    273: 
                    274: Ответ:
                    275: Дзержинский.
                    276: 
                    277: Комментарий:
                    278: Как известно, прозвище Феликса Эдмундовича Дзержинского - "Железный
                    279: Феликс", а атомный номер железа - 26. А еще мы немного намекнули на
                    280: строчки из поэмы "Хорошо!" В.В. Маяковского: "Юноше, обдумывающему
                    281: житье, решающему, делать жизнь с кого, скажу, не задумываясь - делай ее
                    282: с товарища Дзержинского". Вот такая вот железная логика!
                    283: 
                    284: Источник:
                    285:    1. https://ru.wikipedia.org/wiki/Дзержинский,_Феликс_Эдмундович
                    286:    2. https://dic.academic.ru/dic.nsf/dic_wingwords/3503/
                    287: 
                    288: Автор:
                    289: Александр Кудрявцев (Николаев)
                    290: 
                    291: Вопрос 3:
                    292: (pic: 20170775.jpg)
                    293:    Перед вами ироничные принтЫ, разработанные по заказу концерна
                    294: Volkswagen AG [фольксвАген а-гэ]. О важности чего они напоминают
                    295: автомобилистам?
                    296: 
                    297: Ответ:
                    298: Соблюдать дистанцию на дороге.
                    299: 
                    300: Зачет:
                    301: Функция автоматического регулирования дистанции.
                    302: 
                    303: Комментарий:
                    304: "Она умная. Она веселая. Она сексуальная. Она девушка моего брата", -
                    305: сообщают надписи на первой рекламе, рекомендуя держать дистанцию.
                    306: Аналогично и со второй рекламой: обидно терпеть тычки, пинки и грубую
                    307: игру от босса, но приходится всё равно играть и держать дистанцию.
                    308: 
                    309: Источник:
                    310:    1. https://www.adsoftheworld.com/media/print/volkswagen_girlfriend
                    311:    2. https://www.adsoftheworld.com/media/print/volkswagen_boss_1
                    312: 
                    313: Автор:
                    314: Дарья Засинец (Гомель)
                    315: 
                    316: Вопрос 4:
                    317: (pic: 20170776.jpg)
                    318:    Оба пропуска на раздаточной карточке содержат один и тот же набор
                    319: букв в одинаковом порядке. Восстановите любой из пропусков.
                    320: 
                    321: Ответ:
                    322: Дома тильды.
                    323: 
                    324: Зачет:
                    325: До Матильды.
                    326: 
                    327: Комментарий:
                    328: Интерьерные текстильные игрушки ручной работы называются тильдами.
                    329: Тильда - это также название типографского знака в виде волнистой черты,
                    330: на что визуально могла навести поза куклы. Во втором случае речь идет о
                    331: Матильде Кшесинской.
                    332: 
                    333: Источник:
                    334:    1. http://krolikdoma.ru/threads/tildy.117/
                    335:    2. http://strana.jili-bili.ru/blog/jilibilitoys/220.html
                    336:    3. https://ru.wikipedia.org/wiki/Тильда
                    337:    4. https://24smi.org/celebrity/4167-matilda-kshesinskaia.html
                    338: 
                    339: Автор:
                    340: Виктор Мялов (Днепр)
                    341: 
                    342: Вопрос 5:
                    343: (pic: 20170777.jpg)
                    344: 
                    345: Ответ:
                    346: [Антонио] КанОва.
                    347: 
                    348: Комментарий:
                    349: Каждая из скульптур напоминает букву фамилии скульптора Антонио КанОвы -
                    350: итальянского мастера, самые крупные собрания работ которого находятся в
                    351: парижском Лувре и петербургском Эрмитаже. Слова самого вопроса
                    352: неслучайно написаны под соответствующими изображениями: К - кто, А -
                    353: автор. Дублирование изображений показывает совпадение второй и шестой
                    354: букв в фамилии.
                    355: 
                    356: Источник:
                    357:    1. https://ru.wikipedia.org/wiki/Канова,_Антонио
                    358:    2. https://yandex.ru/images/search?text=канова%20скульптуры%20фото&stype=image
                    359: 
                    360: Автор:
                    361: Александр Сидоренков (Смоленск)
                    362: 
                    363: Вопрос 6:
                    364: (pic: 20170778.jpg)
                    365:    Событие, которое изображено на карикатуре, послужило одним из мотивов
                    366: выбора персонажа известным автором. Согласно воспоминаниям, само его имя
                    367: звучало иронически. Назовите этого персонажа.
                    368: 
                    369: Ответ:
                    370: [Эркюль] Пуаро.
                    371: 
                    372: Комментарий:
                    373: Карикатура посвящена позиции Бельгии, которая отказала Германии в
                    374: продвижении ее войск, чем вызвала симпатии в Европе. Присутствие
                    375: большого количества бельгийских эмигрантов в Англии стало одним из
                    376: побудительных мотивов Агаты Кристи в выборе типажа для детектива,
                    377: который должен быть большим профессионалом и знаменитостью, но при этом
                    378: не находиться на государственной службе. Имя Эркюль (Геркулес) никак не
                    379: сочетается с комической внешностью и маленьким ростом Пуаро.
                    380: 
                    381: Источник:
                    382:    1. Коллекция Агаты Кристи. Выпуск N 2 (2016). Загадочное происшествие
                    383: в Стайплзе. - С. 14-15.
                    384:    2. https://en.wikipedia.org/wiki/Belgium_in_World_War_I#German_invasion
                    385: 
                    386: Автор:
                    387: Дмитрий Соловьёв (Нижний Новгород)
                    388: 
                    389: Вопрос 7:
                    390: (pic: 20170779.jpg)
                    391:    Подскажем, что в розданном вам фрагменте таблицы, приведенной в одной
                    392: статье Википедии, на месте пропусков должны стоять имена собственные.
                    393: Возможно, это вам поможет дать максимально точный ответ на вопрос: "Что
                    394: стало красным, начиная с третьей короткометражки этого списка?".
                    395: 
                    396: Ответ:
                    397: Нос Дейла.
                    398: 
                    399: Зачет:
                    400: По смыслу.
                    401: 
                    402: Комментарий:
                    403: На самом деле бурундуки Чип (ПРОПУСК 1) и Дейл (ПРОПУСК 2) появились в
                    404: мультфильмах Уолта Диснея задолго до 1989 года, когда были показаны
                    405: первые серии "Chip 'n Dale: Rescue Rangers" [чип энд дэйл: рЕскью
                    406: рЭйнджерс]. Поначалу они не отличались так сильно друг от друга - даже
                    407: носы были у обоих черные, но уже начиная с третьего мультфильма Дейла
                    408: стали рисовать с красным носом. Слово "поможет" в вопросе и красные
                    409: оконечности названий мультфильмов на раздатке могли послужить небольшими
                    410: подсказками. Ну, или не послужить...
                    411: 
                    412: Источник:
                    413:    1. https://en.wikipedia.org/wiki/Chip_'n'_Dale
                    414:    2. Мультфильм "Chip an' Dale" (1947).
                    415:    3. ЛОАВ.
                    416: 
                    417: Автор:
                    418: Юрий Чулинин (Новосибирск - Поварово)
                    419: 
                    420: Вопрос 8:
                    421: (pic: 20170780.jpg)
                    422:    Можно сказать, что автор вопроса совершил ЭТО. А еще ЭТО - второе
                    423: название... Чего?
                    424: 
                    425: Ответ:
                    426: Щели Кассини.
                    427: 
                    428: Зачет:
                    429: Промежутка между внешними кольцами Сатурна.
                    430: 
                    431: Комментарий:
                    432: ЭТО - деление Кассини (промежуток между внешними кольцами Сатурна
                    433: шириной около 5000 км, открытый в 1675 году итальянским и французским
                    434: астрономом Жаном-Домиником Кассини). На раздатке на фоне Сатурна
                    435: изображен как раз сам астроном, и автор вопроса совершил его "деление".
                    436: Вдобавок диагональная полоса на раздатке еще и сама напоминает знак
                    437: деления.
                    438: 
                    439: Источник:
                    440:    1. http://expedientsecreto.blogspot.com/2012/04/cassini-y-las-lunas-de-saturno.html
                    441:    2. http://ru.wikipedia.org/wiki/Щель_Кассини
                    442: 
                    443: Автор:
                    444: Александр Кудрявцев (Николаев)
                    445: 
                    446: Вопрос 9:
                    447: (pic: 20170781.jpg)
                    448:    На раздатке картина ТоддЕо ГАдди "Обручение Богоматери". Искусствовед
                    449: Борис ВИппер полагает, что действие этого полотна [ПРОПУСК]. В индийской
                    450: мифологии Сита [ПРОПУСК]. Заполните любой из пропусков, неразличимых на
                    451: слух, тремя словами.
                    452: 
                    453: Ответ:
                    454: Выходит за раму.
                    455: 
                    456: Зачет:
                    457: Выходит за Раму.
                    458: 
                    459: Комментарий:
                    460: Фрагменты труб на заднем плане картины без изображений самих музыкантов
                    461: позволили искусствоведу сделать вывод, что сюжетное действие не
                    462: уместилось целиком внутри рамы. СИта - героиня древнеиндийского эпоса
                    463: "РамаЯна" - была, как известно, супругой Рамы, т.е. вышла за него замуж.
                    464: 
                    465: Источник:
                    466:    1. Б.Р. Виппер. Итальянский ренессанс XIII-XVI веков. Том 1.
                    467: http://flibusta.is/b/378539/read
                    468:    2. https://www.wga.hu/html_m/g/gaddi/taddeo/croce/index.html
                    469:    3. https://ru.wikipedia.org/wiki/Сита
                    470: 
                    471: Автор:
                    472: Александр Сидоренков (Смоленск)
                    473: 
                    474: Тур:
                    475: 2.1. "МГТУ - Беспощадные"
                    476: 
                    477: Дата:
                    478: 23-Dec-2017
                    479: 
                    480: Инфо:
                    481: Команда благодарит за тестирование вопросов и ценные замечания игроков
                    482: команды "От винта - Братья по фазе": Дмитрия Башука, Ольгу Неумывакину,
                    483: Максима Евланова (все - Харьков).
                    484: 
                    485: Вопрос 1:
                    486: На карикатуре Гюндуза Агаева люди, словно зомби, уткнувшись в экраны
                    487: мобильных телефонов, уходят под воду. Назовите человека, который на этой
                    488: карикатуре изображен сидящим на лодке.
                    489: 
                    490: Ответ:
                    491: [Марк] Цукерберг.
                    492: 
                    493: Комментарий:
                    494: На этой иллюстрации основатель социальной сети Facebook [фейсбук] Марк
                    495: Цукерберг использует логотип компании - букву f [эф] - в качестве
                    496: флейты, а сам, восседая на подлодке, выступает гамельнским крысоловом,
                    497: уводящим народ на гибель.
                    498: 
                    499: Источник:
                    500: https://kulturologia.ru/blogs/060318/38039/
                    501: 
                    502: Автор:
                    503: Николай Кукшинов (Москва)
                    504: 
                    505: Вопрос 2:
                    506: После получения пятого "Золотого мяча" Криштиану Роналду заявил: "Я -
                    507: лучший футболист в истории мирового футбола". Один из комментариев к
                    508: этой новости на сайте championat.com [чемпионат точка ком] гласит: "ты
                    509: даже не лучший on [он] в истории футбола". Какие буквы мы пропустили в
                    510: тексте вопроса?
                    511: 
                    512: Ответ:
                    513: R, a, l, d, o.
                    514: 
                    515: Зачет:
                    516: Р, а, л, д, о; Р, а, л, д, у.
                    517: 
                    518: Комментарий:
                    519: Не только автор комментария придерживается мнения, что Зубастик лучше
                    520: Криштиану.
                    521: 
                    522: Источник:
                    523: https://www.championat.com/football/news-2974519-ronaldu-ja--luchshij-futbolist-v-istorii-mirovogo-futbola.html
                    524: 
                    525: Автор:
                    526: Николай Кукшинов (Москва)
                    527: 
                    528: Вопрос 3:
                    529: (pic: 20170782.jpg)
                    530:    В 1911 году в Вене было построено здание по проекту Адольфа Лооса.
                    531: Из-за необычного для того времени внешнего вида здание получило прозвище
                    532: "Дом [ПРОПУСК]". Участники эксперимента, проведенного в Массачусетском
                    533: технологическом институте, опознали знаменитостей [ПРОПУСК] всего в 46%
                    534: случаев. Заполните пропуск двумя словами, начинающимися на одну и ту же
                    535: букву.
                    536: 
                    537: Ответ:
                    538: Без бровей.
                    539: 
                    540: Комментарий:
                    541: Модернистское здание Адольфа Лооса из-за отсутствия декора снаружи, в
                    542: частности вокруг окон, получило прозвище "Дом без бровей". Участникам
                    543: эксперимента в MIT [эм-ай-ти] показывали фотографии известных личностей.
                    544: На одной группе фотографий в Фотошопе убрали глаза, на второй - брови. В
                    545: первом случае процент узнавания составил около 59%, во втором - около
                    546: 46%, что говорит о важности бровей в формировании облика человека.
                    547: 
                    548: Источник:
                    549:    1. https://venagid.ru/17942-looshaus
                    550:    2. https://images.adsttc.com/media/images/5850/1189/e58e/cef7/1900/00d3/slideshow/Figura_1._Imagen_Almacenes_Goldmand___Salatsch___en_Michaelerplatz__de_Adolf_Loos._Imagen_de_dominio_publico.jpg
                    551:    3. http://web.mit.edu/sinhalab/Papers/sinha_eyebrows.pdf
                    552: 
                    553: Автор:
                    554: Владимир Бабаев (Моздок - Подольск)
                    555: 
                    556: Вопрос 4:
                    557: Фанатичный коллекционер Шейн Хили иногда собирает по 4-5 и даже 7-8
                    558: разных версий одного экземпляра. Он говорит, что никому не доверит свою
                    559: коллекцию и даже подумывает о НЕМ. Назовите ЕГО двумя словами,
                    560: начинающимися на соседние буквы алфавита.
                    561: 
                    562: Ответ:
                    563: Виниловый гроб.
                    564: 
                    565: Комментарий:
                    566: Шейн Хили - фанат группы "Status Quo" [стАтус кво]. У него нет жены и
                    567: детей, поэтому богатую коллекцию винила он собирается в прямом смысле
                    568: забрать с собой в могилу. Он собирает несколько разных вариантов одной
                    569: записи - например, из-за ошибок на обложке, либо из-за другого
                    570: оформления. Числа в тексте вопроса могли навести на скорость оборота
                    571: виниловых пластинок - 45 и 78 оборотов в минуту.
                    572: 
                    573: Источник:
                    574: Документальный фильм "Музыкальный магазин" (Sound it out) (2011).
                    575: 
                    576: Автор:
                    577: Владимир Бабаев (Моздок - Подольск)
                    578: 
                    579: Вопрос 5:
                    580: (pic: 20170783.jpg)
                    581:    В стихотворении Вадима Шефнера 1957 года говорится о первоначальном
                    582: плане Васильевского острова: как будто Петру I план подкинули ОНИ.
                    583: Возможно, поэту вспомнилась гипотеза 1877 года, опровергнутая почти
                    584: через сто лет. Какое слово мы заменили словом "ОНИ"?
                    585: 
                    586: Ответ:
                    587: Марсиане.
                    588: 
                    589: Комментарий:
                    590: На Васильевском острове изначально задумывали построить сеть судоходных
                    591: каналов. От этой идеи отказались, но две стороны одной улицы так и
                    592: остались разными линиями. На розданной карте внизу можно увидеть
                    593: Масляный канал, что может быть небольшой подсказкой. Строки из
                    594: стихотворения Шефнера: "... Я вижу: лежит он на плане, // В грядущее
                    595: запросто вхож, - // Как будто Петру марсиане // Подбросили этот
                    596: чертеж...". Впервые о марсианских каналах сообщил Джованни Скиапарелли.
                    597: А в 1970-х аппарат "Маринер-9" провел фотосъемку поверхности Марса.
                    598: БОльшая часть каналов оказалась оптической иллюзией, и только некоторые
                    599: совпали с образованиями рельефа.
                    600: 
                    601: Источник:
                    602:    1. В.С. Шефнер. Стихи о Васильевском острове.
                    603: http://flibusta.is/b/484686/read#t130
                    604:    2. https://ru.wikipedia.org/wiki/Васильевский_остров
                    605:    3. https://ru.wikipedia.org/wiki/Марсианские_каналы
                    606: 
                    607: Автор:
                    608: Владимир Бабаев (Моздок - Подольск)
                    609: 
                    610: Вопрос 6:
                    611: В советские времена Игарка была крупным промышленным центром
                    612: Красноярского края. В песне 1950-х годов Александр Городницкий уподобил
                    613: мостовые Игарки АЛЬФАМ. В рассказе Паустовского АЛЬФЫ раздражали
                    614: композитора, поэтому к роялю он пробирался. Назовите АЛЬФЫ двумя
                    615: словами.
                    616: 
                    617: Ответ:
                    618: Скрипучие половицы.
                    619: 
                    620: Зачет:
                    621: Скрипящие половицы.
                    622: 
                    623: Комментарий:
                    624: Игарка была крупным лесоперерабатывающим и экспортирующим центром на
                    625: Енисее. Практически весь город был деревянным, даже мостовые, которые
                    626: Городницкий сравнил со скрипучими половицами. Рассказ Константина
                    627: Паустовского "Скрипучие половицы" повествует об уединенной жизни Петра
                    628: Ильича Чайковского в деревянном доме, где половицы раздражали
                    629: композитора.
                    630: 
                    631: Источник:
                    632:    1. https://ru.wikipedia.org/wiki/Игарка
                    633:    2. А.М. Городницкий. Атланты. Моя кругосветная жизнь.
                    634: http://flibusta.is/b/323596/read
                    635:    3. К.Г. Паустовский. Повесть о лесах.
                    636: http://flibusta.is/b/349920/read
                    637: 
                    638: Автор:
                    639: Владимир Бабаев (Моздок - Подольск)
                    640: 
                    641: Вопрос 7:
                    642: (pic: 20170784.jpg)
                    643:    Она получила американское гражданство в 1939 году. Ему был запрещен
                    644: обратный въезд в США в 1952 году. В их жизни было много увлечений, искра
                    645: появилась и в отношениях их бывших возлюбленных. Назовите этого ее
                    646: возлюбленного, ее соотечественника, у которого был с ней роман.
                    647: 
                    648: Ответ:
                    649: [Эрих Мария] Ремарк.
                    650: 
                    651: Комментарий:
                    652: На фото - Мария Магдалена (Марлен) Дитрих и Чарльз Спенсер Чаплин.
                    653: Ремарк, у которого был роман с Дитрих, впоследствии женился на Полетт
                    654: Годдар, бывшей жене Чаплина. Слова "искра" и "жизнь" - подсказки, намек
                    655: на роман Ремарка "Искра жизни", который был завершен во многом благодаря
                    656: Годдар. Можно сказать, что фраза "был с ней роман" - это про
                    657: "Триумфальную арку", в которой Дитрих являлась прообразом главной
                    658: героини Жоан Маду.
                    659: 
                    660: Источник:
                    661:    1. http://www.etoday.ru/2012/01/rare-photos-of-famous-people-part-one.php
                    662:    2. https://ru.wikipedia.org/wiki/Чаплин,_Чарльз
                    663:    3. https://ru.wikipedia.org/wiki/Ремарк,_Эрих_Мария
                    664: 
                    665: Автор:
                    666: Алексей Казаков (Москва)
                    667: 
                    668: Вопрос 8:
                    669:    <раздатка>
                    670:    Меня Амфитрион послал сюда к Алкмене;
                    671:    Из гавани сюда сегодня прибыл я;
                    672:    Я должен рассказать, событий не тая,
                    673:    О том, что враг разбит, и полный ход сражений.
                    674:    Короче говоря, я - Созий, Дава сын,
                    675:    Арпага брат, умершего в Панде;
                    676:    Муж Клеантиде,
                    677:    Чей нрав я проклинал не раз один.
                    678:    Добавлю: в Фивах раз наказан был плетями,
                    679:    Хоть никому о том не говорил.
                    680:    А также заклеймен публично был врагами
                    681:    За то, что слишком честно жил.
                    682:    </раздатка>
                    683:    Дуплет.
                    684:    1. В пьесе Мольера Юпитер в облике Амфитриона явился к его жене, а
                    685: Меркурий охранял их покой. Вернувшийся Созий, слуга Амфитриона, встретил
                    686: ИКСА и начал сомневаться в себе. "ИКС" - это название повести середины
                    687: XIX века. Что мы заменили словом "ИКС"?
                    688:    2. Меркурий не впустил в дом Созия. Рассказывая о происшествии
                    689: Амфитриону, Созий говорит, что СДЕЛАЛ ЭТО. Назовите персонажа
                    690: произведения первой половины XIX века, якобы совершившего подобное
                    691: действие.
                    692: 
                    693: Ответ:
                    694:    1. Двойник.
                    695:    2. Унтер-офицерская вдова.
                    696: 
                    697: Комментарий:
                    698:    1. Меркурий принял облик слуги Амфитриона и, рассказав малоизвестные
                    699: факты из жизни Созия, а также поработав кулаками, почти убедил того, что
                    700: является настоящим Созием. "Двойник" - это повесть Достоевского 1846
                    701: года.
                    702:    2. Рассказывая о происшедшем хозяину, Созий говорит, что сам себя
                    703: поколотил. Похожий поступок приписывался унтер-офицерской вдове в пьесе
                    704: Гоголя "Ревизор". Созию, как можно понять из раздаточного материала,
                    705: ранее тоже приходилось несладко. Кстати, слова на раздаточном материале
                    706: произносит Меркурий в образе Созия.
                    707: 
                    708: Источник:
                    709:    1. Ж.-Б. Мольер. Амфитрион. http://flibusta.is/b/37882/read
                    710:    2. https://ru.wikipedia.org/wiki/Двойник_(повесть_Достоевского)
                    711:    3. https://dic.academic.ru/dic.nsf/dic_wingwords/3595/
                    712: 
                    713: Автор:
                    714: Владимир Бабаев (Моздок - Подольск)
                    715: 
                    716: Вопрос 9:
                    717: Джеймс Джойс, описывая море с прибрежными водорослями, образно называет
                    718: его ИКСОМ луны. Центр Жоржа Помпиду в Париже, в котором переплетаются
                    719: воедино различные культурные направления, из-за внешнего вида также
                    720: называют ИКСОМ. Назовите ИКС двумя словами, начинающимися на соседние
                    721: буквы алфавита.
                    722: 
                    723: Ответ:
                    724: Ткацкий станок.
                    725: 
                    726: Комментарий:
                    727: Колеблющиеся вперед-назад водоросли Джойс уподобил нитям, а всю картину
                    728: назвал ткацким станком луны. Здание Центра Жоржа Помпиду в стиле хайтек
                    729: получило несколько иронических прозвищ, среди которых "пароход",
                    730: "газометр" и "ткацкий станок".
                    731: 
                    732: Источник:
                    733:    1. Дж. Джойс. Улисс.
                    734: https://books.google.ru/books?id=P1YFBAAAQBAJ&pg=PT42#v=onepage&q&f=false
                    735:    2. Э. Пастре, П. Экерлин, Й. Циприк. Париж: Путеводитель.
                    736: https://books.google.ru/books?id=LTTMAAAAQBAJ&pg=PA40#v=onepage&q&f=false
                    737:    3. http://www.liveinternet.ru/users/muza-105/post343045964/
                    738: 
                    739: Автор:
                    740: Владимир Бабаев (Моздок - Подольск)
                    741: 
                    742: Тур:
                    743: 2.2. "МГТУ - Мотыга"
                    744: 
                    745: Дата:
                    746: 23-Dec-2017
                    747: 
                    748: Инфо:
                    749: Команда благодарит за помощь в создании тура, за тестирование вопросов и
                    750: ценные замечания Дмитрия Башука и игроков команды "От винта - Братья по
                    751: фазе": Вадима Данько, Ольгу Неумывакину, Максима Евланова (все -
                    752: Харьков), а также Вячеслава Говорухина (Нахабино), Якова Кушнира
                    753: (Москва), Любовь Бадикову (Балашиха), Марию Коврижкину (Туймазы -
                    754: Видное), Алексея Каплиёва (Ногинск) и Романа Петракова (Томск - Москва).
                    755: 
                    756: Вопрос 1:
                    757: В песне группы "Самбатион" описывается рецепт блюда, среди прочих
                    758: ингредиентов которого упоминаются лук, морковь и барбарис. Рефреном этой
                    759: песни является фраза, которая последним словом немного отличается от
                    760: названия произведения 1967 года. Напишите эту фразу.
                    761: 
                    762: Ответ:
                    763: All you need is plov.
                    764: 
                    765: Зачет:
                    766: С незначительными отклонениями.
                    767: 
                    768: Комментарий:
                    769: Песня называется "Давай делать плов", а остальные ингредиенты - мясо,
                    770: рис и зира. Вам могла помочь рифма "рис - барбарис", однако в песне эти
                    771: слова не рифмуются. Произведение 1967 года - песня группы "Beatles" "All
                    772: you need is love".
                    773: 
                    774: Источник:
                    775:    1. https://www.youtube.com/watch?v=ULdvh0aZOZI
                    776:    2. https://ru.wikipedia.org/wiki/All_You_Need_Is_Love
                    777: 
                    778: Автор:
                    779: Дмитрий Пономарёв (Киров - Москва)
                    780: 
                    781: Вопрос 2:
                    782: Персонаж известного романа говорит, что дом болен, имея в виду
                    783: отсутствие в нем маленькой героини. В одном из слов предыдущего
                    784: предложения мы пропустили пять букв. Восстановите слово-неологизм в
                    785: исходном виде.
                    786: 
                    787: Ответ:
                    788: Безлолитен.
                    789: 
                    790: Источник:
                    791: В.В. Набоков. Лолита. http://flibusta.is/b/425308/read
                    792: 
                    793: Автор:
                    794: Анита Корчагина (Раменское)
                    795: 
                    796: Вопрос 3:
                    797: В шуточной викторине под названием "ЭТО или смерть" игрокам предлагается
                    798: угадать, что перед ними: название death-metal [дет-метал] группы или
                    799: наименование продукции. Назовите ЭТО одним словом.
                    800: 
                    801: Ответ:
                    802: IKEA.
                    803: 
                    804: Зачет:
                    805: ИКЕА.
                    806: 
                    807: Комментарий:
                    808: Викторина представляет игроку название предмета мебели или группы, а
                    809: участник должен определить, чем это является на самом деле. Креативщики
                    810: пришли к созданию этого теста после просмотра каталогов ИКЕА, заметив,
                    811: что многие наименования и серии мебели довольно схожи, а иногда и вовсе
                    812: повторяют названия популярных музыкальных групп в стиле death-metal. К
                    813: тому же Скандинавия имеет самую развитую и разнообразную метал-сцену на
                    814: планете.
                    815: 
                    816: Источник:
                    817: http://ikeaordeath.com/
                    818: 
                    819: Автор:
                    820: Анита Корчагина (Раменское)
                    821: 
                    822: Вопрос 4:
                    823: В 2006 году у НИХ возникли финансовые проблемы при транспортировке
                    824: пиротехники в Грецию. Однако это не помешало ИМ достичь показателя 292,
                    825: рекордного на тот момент. Назовите ИХ.
                    826: 
                    827: Ответ:
                    828: "Lordi".
                    829: 
                    830: Зачет:
                    831: "Лорди".
                    832: 
                    833: Комментарий:
                    834: На Евровидении в 2006 году, прошедшем в Греции, Финляндия одержала
                    835: первую победу за 45 лет своего участия, ее представляла хард-рок-группа
                    836: Lordi, которая набрала 292 очка. Группа знаменита тем, что выступает в
                    837: масках и костюмах монстров, а их выступления зачастую сопровождаются
                    838: яркими пиротехническими шоу. Участие группы на Евровидении вызвало
                    839: скандал, некоторые общественные организации назвали группу "сатанинской"
                    840: за имидж и тексты. Кроме того, у "Lordi" возникли финансовые проблемы с
                    841: доставкой дорогостоящей аппаратуры и пиротехники, но благодаря помощи
                    842: спонсоров эта проблема была решена.
                    843: 
                    844: Источник:
                    845: https://ru.wikipedia.org/wiki/Lordi
                    846: 
                    847: Автор:
                    848: Дмитрий Пономарёв (Киров - Москва)
                    849: 
                    850: Вопрос 5:
                    851: (pic: 20170785.jpg)
                    852:    На раздаточном материале схематично показан процесс изготовления ЕЕ.
                    853: Назовите ЕЕ двумя словами.
                    854: 
                    855: Ответ:
                    856: Коктейльная вишня.
                    857: 
                    858: Зачет:
                    859: Мараскиновая вишня/вишенка.
                    860: 
                    861: Комментарий:
                    862: Перед вами сложный технологический процесс приготовления коктейльной
                    863: вишни, которую используют в том числе и для украшения тОрта, в виде
                    864: которого представлена схема на раздаточном материале. Диоксид серы и
                    865: негашеная известь используются для уплотнения вишни, вымачивание в воде
                    866: необходимо для удаления обесцвечивающих агентов, бисульфит натрия
                    867: придает плотную структуру.
                    868: 
                    869: Источник:
                    870: https://ru.wikipedia.org/wiki/Коктейльная_вишня
                    871: 
                    872: Автор:
                    873: Дмитрий Пономарёв (Киров - Москва)
                    874: 
                    875: Вопрос 6:
                    876: (pic: 20170786.jpg)
                    877:    Король Луи Филипп I обещал избегать как консервативного, так и
                    878: радикально левого подхода. Эту политику называют словосочетанием,
                    879: которое можно перевести на русский язык как "ТАКАЯ ОНА". Можно сказать,
                    880: что на раздаточном материале мы закрыли черным прямоугольником ТАКУЮ ЕЕ.
                    881: Что мы заменили ТАКОЙ ЕЮ?
                    882: 
                    883: Ответ:
                    884: Золотая середина.
                    885: 
                    886: Комментарий:
                    887: Juste milieu [жюст мильё] - термин, который использовался для описания
                    888: центристских политических философий, в том числе и во время Французской
                    889: Июльской монархии при Луи Филиппе I. А на раздаточном материале золото
                    890: располагается в середине.
                    891: 
                    892: Источник:
                    893:    1. https://en.wikipedia.org/wiki/July_Monarchy
                    894:    2. http://context.reverso.net/перевод/французский-русский/juste+milieu
                    895: 
                    896: Автор:
                    897: Дмитрий Пономарёв (Киров - Москва)
                    898: 
                    899: Вопрос 7:
                    900: Герой Дмитрия Глуховского сравнивает летнюю Москву с НЕЙ, упоминая
                    901: медленное движение по Третьему кольцу, по Садовому кольцу и по Кольцевой
                    902: линии метро. Назовите ЕЕ.
                    903: 
                    904: Ответ:
                    905: Микроволновка.
                    906: 
                    907: Зачет:
                    908: Микроволновая печь; СВЧ-печь.
                    909: 
                    910: Источник:
                    911: Д.А. Глуховский. Текст.
                    912: https://books.google.ru/books?id=73cqDwAAQBAJ&pg=PT37#v=onepage&q&f=false
                    913: 
                    914: Автор:
                    915: Любовь Бадикова (Балашиха), Дмитрий Пономарёв (Киров - Москва)
                    916: 
                    917: Вопрос 8:
                    918:    <раздатка>
                    919:    villa, Volvo, vovve
                    920:    </раздатка>
                    921:    Перед вами выражение о составляющих "шведской мечты". Прочитайте его
                    922: и напишите перевод третьего слова.
                    923: 
                    924: Ответ:
                    925: Собака.
                    926: 
                    927: Зачет:
                    928: Пес.
                    929: 
                    930: Комментарий:
                    931: А всё вместе переводится как "дом, Вольво, собака". Слово "vovve" на
                    932: слух похоже на лай собаки.
                    933: 
                    934: Источник:
                    935:    1. https://www.vokrugsveta.ua/interesting/10-mifov-o-shvetsii-06-06-2017
                    936:    2. https://en.wiktionary.org/wiki/vovve
                    937: 
                    938: Автор:
                    939: Дмитрий Гасилин (Москва), Дмитрий Пономарёв (Киров - Москва)
                    940: 
                    941: Вопрос 9:
                    942: Внимание, в вопросе есть замена.
                    943:    Карлес Пуйоль быстро восстанавливался после травм, в связи с чем про
                    944: него говорили, что если Карлес СДЕЛАЕТ ЭТО, то пропустит максимум две
                    945: недели. Что НЕ СДЕЛАЕТ ЭТОГО в названии произведения 1997 года?
                    946: 
                    947: Ответ:
                    948: Завтра.
                    949: 
                    950: Комментарий:
                    951: Даже тяжелые травмы быстро заживали на бывшем капитане "Барселоны"
                    952: Карлосе Пуйоле, в связи с чем и был придуман этот анекдот. Произведение
                    953: 1997 года - один из фильмов бондианы "Завтра не умрет никогда".
                    954: 
                    955: Источник:
                    956:    1. http://sport.bigmir.net/football/other/1561355-Konec-sveta--Shest--futbolistov--kotorye-ego-tochno-perezhivut
                    957:    2. https://ru.wikipedia.org/wiki/Завтра_не_умрёт_никогда
                    958: 
                    959: Автор:
                    960: Павел Голубев (Москва)
                    961: 
                    962: Тур:
                    963: 3.1. "Лимонад"
                    964: 
                    965: Дата:
                    966: 23-Dec-2017
                    967: 
                    968: Редактор:
                    969: Серафим Шибанов (Москва)
                    970: 
                    971: Вопрос 1:
                    972: Есть мнение, что СССР отложил начало войны с Финляндией на начало зимы в
                    973: том числе потому, что осенью животные были неконтролируемы и ОНА
                    974: действовать не могла. Назовите ЕЕ двумя словами, начинающимися на
                    975: соседние буквы алфавита.
                    976: 
                    977: Ответ:
                    978: Лосиная кавалерия.
                    979: 
                    980: Комментарий:
                    981: Некоторые источники утверждают, что в тридцатые годы в СССР было
                    982: подготовлено более полутора тысяч боевых лосей. А на рогах, говорят,
                    983: можно удобно пристроить пулемет.
                    984: 
                    985: Источник:
                    986: https://www.popmech.ru/weapon/10162-rogataya-kavaleriya-boevye-losi/
                    987: 
                    988: Автор:
                    989: Сергей Сыромятников (Москва)
                    990: 
                    991: Вопрос 2:
                    992: Внимание, в вопросе есть замены.
                    993:    В Европе рубежа XIX-XX веков газеты рекламировали средства для
                    994: ускоренного роста бороды, а многие люди без всякой надобности носили
                    995: очки. Анна Чайковская объясняет это тем, что ПЕРВАЯ часто считалась
                    996: недостатком, а ВТОРАЯ - достоинством. Можно сказать, что ПЕРВАЯ и ВТОРАЯ
                    997: ассоциируются с известным европейским коллективом. Из какого города?
                    998: 
                    999: Ответ:
                   1000: Турин.
                   1001: 
                   1002: Комментарий:
                   1003: ПЕРВАЯ - молодость, ВТОРАЯ - старость. Слово "Ювентус" переводится как
                   1004: "молодость", но в то же время этот клуб называют "старой синьорой".
                   1005: 
                   1006: Источник:
                   1007:    1. https://anna-bpguide.livejournal.com/488662.html
                   1008:    2. https://ru.wikipedia.org/wiki/Ювентус
                   1009: 
                   1010: Автор:
                   1011: Игорь Бронштейн (Тарко-Сале)
                   1012: 
                   1013: Вопрос 3:
                   1014: Один интернет-пользователь предлагает радикально изменить систему линий
                   1015: московского метро. В частности, на предложенной этим пользователем схеме
                   1016: можно увидеть ЕГО. Другое "ОНО" является частью комплекса, созданного по
                   1017: инициативе ленинградского поэта Михаила Дудина. Назовите ЕГО двумя
                   1018: словами.
                   1019: 
                   1020: Ответ:
                   1021: Разорванное кольцо.
                   1022: 
                   1023: Комментарий:
                   1024: Метрофанат предложил схему, на которой отсутствует привычная нам
                   1025: замкнутая Кольцевая линия, а ее части входят в состав других линий.
                   1026: Мемориал "Разорванное кольцо" находится на берегу Ладожского озера, в
                   1027: месте, где к нему выходила разорвавшая кольцо блокады "Дорога жизни".
                   1028: 
                   1029: Источник:
                   1030:    1. http://forum.nashtransport.ru/blogs/metrofuture/index.php?showentry=6608
                   1031:    2. https://ru.wikipedia.org/wiki/Разорванное_кольцо
                   1032:    3. https://ru.wikipedia.org/wiki/Зелёный_пояс_Славы
                   1033: 
                   1034: Автор:
                   1035: Игорь Бронштейн (Тарко-Сале)
                   1036: 
                   1037: Вопрос 4:
                   1038: Внимание, слово "АЛЬФА" в вопросе заменяет два слова.
                   1039:    Героиню одной сказки заподозрили в том, что она не принцесса, потому
                   1040: что у этой героини была АЛЬФА. Чем спустя непродолжительное время
                   1041: становится АЛЬФА, согласно источнику конца 1980-х годов?
                   1042: 
                   1043: Ответ:
                   1044: [Просто] землей.
                   1045: 
                   1046: Комментарий:
                   1047: Принцесса в фильме-сказке "Каин XVIII" в присутствии короля убивает
                   1048: комара. Комар оказался просто комаром (а не бомбой), и после его
                   1049: убийства проступила АЛЬФА, т.е. красная кровь. А если верить Виктору
                   1050: Цою, "красная кровь - через час уже просто земля".
                   1051: 
                   1052: Источник:
                   1053:    1. Х/ф "Каин XVIII" (1963), реж. Надежда Кошеверова, Михаил Шапиро,
                   1054: 64-я минута.
                   1055:    2. https://music.yandex.ru/album/10030/track/105324
                   1056: 
                   1057: Автор:
                   1058: Сергей Сыромятников (Москва)
                   1059: 
                   1060: Вопрос 5:
                   1061: (pic: 20170787.jpg)
                   1062:    Основываясь на мнении специалистов, Александр Шапиро полагает, что в
                   1063: композиции "День и ночь" одна из статуй символизирует не ночь, а смерть.
                   1064: Назовите этих специалистов словом, в котором содержится три одинаковых
                   1065: согласных.
                   1066: 
                   1067: Ответ:
                   1068: Маммологи.
                   1069: 
                   1070: Комментарий:
                   1071: Утверждается, что у этой женской фигуры работы Микеланджело есть явные
                   1072: признака рака груди.
                   1073: 
                   1074: Источник:
                   1075: А. Шапиро. Загадки старых мастеров. http://flibusta.is/b/497985/read
                   1076: 
                   1077: Автор:
                   1078: Людмила Батуева (Москва)
                   1079: 
                   1080: Вопрос 6:
                   1081: Внимание, в вопросе есть замены.
                   1082:    К востоку от известного сооружения в XVIII веке массово жили кузнецы.
                   1083: Именно этим городская легенда объясняет тот факт, что на запад стремятся
                   1084: образованные, а на восток - необразованные. Назовите упомянутое
                   1085: сооружение.
                   1086: 
                   1087: Ответ:
                   1088: Аничков мост.
                   1089: 
                   1090: Комментарий:
                   1091: По легенде, в XVIII веке в районе Литейного проспекта и Кузнечного
                   1092: переулка располагались литейные мастерские и кузницы. Поэтому
                   1093: подкованные кони "скачут" от кузниц, т.е. примерно на запад, а
                   1094: неподкованные - наоборот, на восток. Слово "подкованные" мы заменили
                   1095: словом "образованные".
                   1096: 
                   1097: Источник:
                   1098: https://pantv.livejournal.com/285711.html
                   1099: 
                   1100: Автор:
                   1101: Сергей Сыромятников (Москва)
                   1102: 
                   1103: Вопрос 7:
                   1104: Внимание, в вопросе есть замены.
                   1105:    Великая княгиня Елизавета Федоровна после убийства мужа решила
                   1106: посвятить себя Богу и, можно сказать, отказалась от ПЕРВОГО в пользу
                   1107: ВТОРОЙ. Назовите женщину, обвиненную в неспособности выбрать между
                   1108: ПЕРВЫМ и ВТОРОЙ.
                   1109: 
                   1110: Ответ:
                   1111: [Анна] Ахматова.
                   1112: 
                   1113: Комментарий:
                   1114: ПЕРВЫЙ - будуар, ВТОРАЯ - молельня. Жданов назвал Ахматову "взбесившейся
                   1115: барынькой, мечущейся между будуаром и молельней".
                   1116: 
                   1117: Источник:
                   1118:    1. http://www.vesti.ru/doc.html?id=256761
                   1119:    2. https://ru.wikipedia.org/wiki/Постановление_оргбюро_ЦК_ВКП(б)_%C2%ABО_журналах_%E2%80%9EЗвезда%E2%80%9C_и_%E2%80%9EЛенинград%E2%80%9C%C2%BB
                   1120: 
                   1121: Автор:
                   1122: Сергей Сыромятников (Москва)
                   1123: 
                   1124: Вопрос 8:
                   1125: На олимпийском турнире по гольфу 2016 года попадание в 18-ю лунку было
                   1126: особенно трудным. Спортсменкам было сложно рассчитать направление
                   1127: отскока мяча после приземления. Комментатор сравнил окрестность этой
                   1128: лунки с НЕЙ. Назовите ЕЕ.
                   1129: 
                   1130: Ответ:
                   1131: Москва.
                   1132: 
                   1133: Комментарий:
                   1134: По мнению комментатора, лунка была, как Москва, на семи холмах.
                   1135: 
                   1136: Источник:
                   1137: Репортаж с финала женского олимпийского турнира по гольфу.
                   1138: 
                   1139: Автор:
                   1140: Сергей Сыромятников (Москва)
                   1141: 
                   1142: Вопрос 9:
                   1143: Внимание, в вопросе есть замена.
                   1144:    Израильский драматург Рои Хен переводил на иврит классическое
                   1145: произведение русской литературы и столкнулся с тем, что в иврите нет
                   1146: аналога слова "ПАРАДОКС". Хен пишет, что своего рода ПАРАДОКСОМ можно
                   1147: назвать великого раввина, а заглавный герой переводимого произведения -
                   1148: всего лишь очень талантливый мальчик. О каком произведении идет речь?
                   1149: 
                   1150: Ответ:
                   1151: "Моцарт и Сальери".
                   1152: 
                   1153: Комментарий:
                   1154: Словом "ПАРАДОКС" мы заменили слово "гений". Гений, если верить тому же
                   1155: Пушкину, "парадоксов друг". Хен говорит, что среди евреев нет гениев,
                   1156: зато есть талантливые мальчики.
                   1157: 
                   1158: Источник:
                   1159: http://www.colta.ru/articles/literature/13866
                   1160: 
                   1161: Автор:
                   1162: Сергей Сыромятников (Москва)
                   1163: 
                   1164: Тур:
                   1165: 3.2. "БорисКо"
                   1166: 
                   1167: Дата:
                   1168: 23-Dec-2017
                   1169: 
1.2       rubashki 1170: Редактор:
                   1171: Серафим Шибанов (Москва)
                   1172: 
1.1       rubashki 1173: Вопрос 1:
                   1174: В шорской народной сказке выдра уклоняется от ежегодной дани духам воды
                   1175: и земли, говоря духу земли, что живет в воде, а духу воды - наоборот.
                   1176: Сюжет этой сказки имеет много общего с подмосковной легендой о
                   1177: происхождении названия города. Назовите глагол, от которого, согласно
                   1178: этой легенде, произошло название города.
                   1179: 
                   1180: Ответ:
                   1181: Объегорить.
                   1182: 
                   1183: Комментарий:
                   1184: По одной из версий, местные жители схожим образом уклонялись от выплаты
                   1185: дани московским, рязанским и владимирским князьям.
                   1186: 
                   1187: Источник:
                   1188:    1. http://www.rodon.org/other/shs.htm
                   1189:    2. http://www.moskvaobl.ru/bcity/2337/
                   1190: 
                   1191: Автор:
                   1192: Василий Сумин (Егорьевск), в редакции Серафима Шибанова (Москва)
                   1193: 
                   1194: Вопрос 2:
                   1195:    <раздатка>
                   1196:    В / А
                   1197:    </раздатка>
                   1198:    Название какого российского города мы так зашифровали?
                   1199: 
                   1200: Ответ:
                   1201: Омск.
                   1202: 
                   1203: Автор:
                   1204: Василий Сумин (Егорьевск)
                   1205: 
                   1206: Вопрос 3:
                   1207: По воспоминаниям сестры игрока сборной США, после "Чуда на льду" она не
                   1208: видела такого количества флагов на улицах с 1960-х годов. Но в 1960-е с
                   1209: ними ДЕЛАЛИ ЭТО, замечает она. Герой какого произведения СДЕЛАЛ ЭТО с
                   1210: последним мостом?
                   1211: 
                   1212: Ответ:
                   1213: "Я свободен".
                   1214: 
                   1215: Комментарий:
                   1216: В 1960-е годы в США из-за сложной политической обстановки происходили
                   1217: народные выступления, на которых жгли национальные флаги. Герой песни "Я
                   1218: свободен" сжигает последний мост "под холодный шепот звезд".
                   1219: 
                   1220: Источник:
                   1221:    1. https://ru.wikipedia.org/wiki/Чудо_на_льду
                   1222:    2. https://music.yandex.ru/album/63576/track/595258
                   1223: 
                   1224: Автор:
                   1225: Василий Сумин (Егорьевск)
                   1226: 
                   1227: Вопрос 4:
                   1228: Реклама оружейного магазина в компьютерной игре "GTA V" предлагает во
                   1229: время зомби-апокалипсиса не звонить 911, а воспользоваться другими
                   1230: цифрами. Напишите эти три цифры.
                   1231: 
                   1232: Ответ:
                   1233: 357.
                   1234: 
                   1235: Комментарий:
                   1236: .357 Магнум - американский револьверный патрон большой мощности.
                   1237: 
                   1238: Источник:
                   1239: Компьютерная игра "GTA V".
                   1240: 
                   1241: Автор:
                   1242: Василий Сумин (Егорьевск)
                   1243: 
                   1244: Вопрос 5:
                   1245: В романе Сергея Лукьяненко "Дневной Дозор" описывается старая фотография
                   1246: Завулона на фоне Крайст-Чёрч. Сам Завулон замечает, что было трудно
                   1247: уговорить "этого чопорного поэтического сухаря" потратить время не на
                   1248: НЕЕ, а на собственного студента. Назовите ЕЕ.
                   1249: 
                   1250: Ответ:
                   1251: Алиса [Лидделл].
                   1252: 
                   1253: Комментарий:
                   1254: На фото - Завулон в студенческой одежде студента Оксфорда. Льюис Кэрролл
                   1255: преподавал математику в оксфордском колледже Крайст-Чёрч.
1.3     ! rubashki 1256:    z-checkdb: В романе упоминается "маленькая девочка", без указания
        !          1257: имени, а Алиса Лидделл - далеко не единственная маленькая девочка,
        !          1258: которую фотографировал Кэрролл, см.
        !          1259: https://commons.wikimedia.org/wiki/Category:Photographs_of_children_by_Lewis_Carroll
        !          1260: (Антон Икрянников).
1.1       rubashki 1261: 
                   1262: Источник:
                   1263: С.В. Лукьяненко, В.Н. Васильев. Дневной Дозор.
                   1264: http://www.rusf.ru/lukian/books/dnevnoy_dozor/dnevnoy_dozor_1_02.htm
                   1265: 
                   1266: Автор:
                   1267: Олег Евстафьев (Ступино)
                   1268: 
                   1269: Вопрос 6:
                   1270:    <раздатка>
                   1271:    Лучшие уходят
                   1272:    Вот и ...
                   1273:    Тише брат не бейся
                   1274:    Тише . . .
                   1275:    </раздатка>
                   1276:    В этом четверостишии, написанном недели две назад, мы не только
                   1277: сделали два пропуска, которые очень похожи, но и слегка изменили одно из
                   1278: слов. Воспроизведите любой из пропусков.
                   1279: 
                   1280: Ответ:
                   1281: Броневой.
                   1282: 
                   1283: Зачет:
                   1284: Бро не вой.
                   1285: 
                   1286: Комментарий:
                   1287: В третьей строке было "бро", а не "брат".
                   1288: 
                   1289: Источник:
                   1290: https://vk.com/depressyashki?w=wall-95488043_81594
                   1291: 
                   1292: Автор:
                   1293: Олег Евстафьев (Ступино)
                   1294: 
                   1295: Вопрос 7:
                   1296: В одном из выпусков "Московского комсомольца" в январе 2013 года
                   1297: обсуждался возможный переход в московский "Спартак" двух полузащитников
                   1298: сборной России. Статья называлась "ИКСОВ или ИКС?". Какое имя мы
                   1299: обозначили как ИКС?
                   1300: 
                   1301: Ответ:
                   1302: Денис.
                   1303: 
                   1304: Комментарий:
                   1305: Речь шла об Игоре Денисове и Денисе Глушакове. Кстати, оба так и не
                   1306: пришли в команду. :-)
                   1307: 
                   1308: Источник:
                   1309: http://www.mk.ru/sport/2013/01/10/796418-denisov-ili-denis.html
                   1310: 
                   1311: Автор:
                   1312: Борис Скоморохов (Ступино), в редакции Серафима Шибанова (Москва)
                   1313: 
                   1314: Вопрос 8:
                   1315: Три крупнейших испаноязычных телеканала США объявили, что больше не
                   1316: будут приглашать для съемок в сериалах ТАКИХ актеров. Да и вообще ТАКИХ
                   1317: людей нельзя показывать по телевидению. Ведь, согласно последним данным
                   1318: психологии, ТАКИЕ люди, "хотят что-то скрыть от окружающих". Какие -
                   1319: ТАКИЕ?
                   1320: 
                   1321: Ответ:
                   1322: Носящие усы и/или бороды.
                   1323: 
                   1324: Зачет:
                   1325: По упоминанию усов и/или бороды.
                   1326: 
                   1327: Источник:
                   1328: Газета "Комсомольская правда", 16.04.2004 г. - С. 20.
                   1329: 
                   1330: Автор:
                   1331: Борис Скоморохов (Ступино), в редакции Серафима Шибанова (Москва)
                   1332: 
                   1333: Вопрос 9:
                   1334: В финальной игре одного из сезонов было сказано, что ЭТО ШОУ -
                   1335: единственное место, где отличники мечтают остаться на второй год. О
                   1336: каком шоу идет речь?
                   1337: 
                   1338: Ответ:
                   1339: КВН.
                   1340: 
                   1341: Комментарий:
                   1342: В КВН чемпион пропускает следующий сезон. Чемпионом обычно становится
                   1343: тот, кто получит больше пятерок. А играть хотят все...
                   1344: 
                   1345: Источник:
                   1346: КВН-2009. Высшая лига. Финал. Музыкальное домашнее задание.
                   1347: "БАК-соучастники". https://www.youtube.com/watch?v=rowduevaBX8&t=9m22s
                   1348: 
                   1349: Автор:
                   1350: Борис Скоморохов (Ступино), в редакции Серафима Шибанова (Москва)
                   1351: 
                   1352: Тур:
                   1353: 4.1. "Зомби-ForУм"
                   1354: 
                   1355: Дата:
                   1356: 24-Feb-2018
                   1357: 
                   1358: Редактор:
                   1359: Евгений Ярков (Тюмень)
                   1360: 
                   1361: Вопрос 1:
                   1362: ОНА - название сайта, приносящего новости литературного мира. Михаил
                   1363: ОсоргИн, описывая одно высокопоставленное лицо конца XVIII века,
                   1364: сравнивает ИХ с центральной батареей и пушками по бокам. Назовите ЕЕ
                   1365: одним словом.
                   1366: 
                   1367: Ответ:
                   1368: Букля.
                   1369: 
                   1370: Комментарий:
                   1371: Букля - сова Гарри Поттера, приносящая ему письма. Тут очевидно
                   1372: использование корня "book" - книга, в названии сайта и отсылка к этой
                   1373: сове. Букли - специальные локоны - набирали свою популярность при
                   1374: Екатерине II, а при Павле I были введены в армию.
                   1375: 
                   1376: Источник:
                   1377:    1. https://buklya.com/
                   1378:    2. М.А. Осоргин. Старинные рассказы.
                   1379: http://az.lib.ru/o/osorgin_m_a/text_0160.shtml
                   1380: 
                   1381: Автор:
                   1382: Евгений Ярков (Тюмень)
                   1383: 
                   1384: Вопрос 2:
                   1385: Мадемуазель Марс требовала, чтобы после ЕГО ухода в комнате открывали
                   1386: окна. Шарль Гривель, называл ЕГО "человеком со ста головами" и
                   1387: аутсайдером. Сам ОН по этому поводу упоминал смешной ореол волос вокруг
                   1388: головы. Уже в наше время, когда ЕГО сыграл Жерар Депардье, многие
                   1389: французы возмущались. Назовите ЕГО.
                   1390: 
                   1391: Ответ:
                   1392: [Александр] Дюма.
                   1393: 
                   1394: Комментарий:
                   1395: Выходец из Гаити, Дюма плохо воспринимался во Франции, несмотря на свой
                   1396: талант.
                   1397: 
                   1398: Источник:
                   1399: М. Чертанов. Дюма. http://flibusta.is/b/389778/read
                   1400: 
                   1401: Автор:
                   1402: Евгений Ярков (Тюмень)
                   1403: 
                   1404: Вопрос 3:
                   1405: Съемки ситкома "Теория большого взрыва" имеют одну особенность. Отмечая
                   1406: эту особенность, сценарист Чак Лорри говорит, что ему часто приходится
                   1407: переписывать текст, а актриса Кейли Куоко сравнивает себя со звездой на
                   1408: рок-концерте. Из-за этой особенности создатели сериала не используют
                   1409: ЕГО. Назовите ЕГО двумя словами, начинающимися на парные согласные.
                   1410: 
                   1411: Ответ:
                   1412: Записанный смех.
                   1413: 
                   1414: Зачет:
                   1415: Закадровый смех и т.п. по смыслу.
                   1416: 
                   1417: Комментарий:
                   1418: Сериал записывается при живой аудитории. Впоследствии смех на отснятый
                   1419: материал не накладывается.
                   1420: 
                   1421: Источник:
                   1422: Э. Рикман. Сериал "Теория Большого взрыва" от А до Я.
                   1423: http://flibusta.is/b/343312/read
                   1424: 
                   1425: Автор:
                   1426: Евгений Ярков (Тюмень)
                   1427: 
                   1428: Вопрос 4:
                   1429: Так называемые дома-гвозди - это постройки, хозяева которых противостоят
                   1430: застройщикам крупных объектов из-за чрезмерно низкой компенсации или по
                   1431: личным причинам. По мнению "Лос-Анджелес Таймс", ЭТОТ ФИЛЬМ был запрещен
                   1432: в Китае из-за опасности массового появления домов-гвоздей. В статье
                   1433: Википедии об ЭТОМ ФИЛЬМЕ можно увидеть главного героя древнеиндийского
                   1434: эпоса. Назовите ЭТОТ ФИЛЬМ.
                   1435: 
                   1436: Ответ:
                   1437: Аватар.
                   1438: 
                   1439: Комментарий:
                   1440: В фильме "Аватар" дом-древо располагается на месторождении ценного
                   1441: ресурса. "Рамаяна" - эпос о Раме - аватаре Вишну, который своим видом
                   1442: напоминает представителя Нави.
                   1443: 
                   1444: Источник:
                   1445:    1. https://ru.wikipedia.org/wiki/Дом-гвоздь
                   1446:    2. https://ru.wikipedia.org/wiki/Аватар_(фильм,_2009)
                   1447:    3. https://ru.wikipedia.org/wiki/Рама
                   1448: 
                   1449: Автор:
                   1450: Евгений Ярков (Тюмень)
                   1451: 
                   1452: Вопрос 5:
                   1453: Архитектор церкви Сант-Иво-алла-Сапиенца, получив заказ, обнаружил, что
                   1454: пространства для будущей церкви очень мало. Елена Джеро пишет, что
                   1455: найденное им нестандартное решение отсылает к геральдическому знаку
                   1456: заказчика Барберини, на котором изображены ОНИ. Среди НИХ есть и
                   1457: плотники. Назовите ИХ.
                   1458: 
                   1459: Ответ:
                   1460: Пчелы.
                   1461: 
                   1462: Комментарий:
                   1463: Для заполнения плоскости шестиугольник требует наименьшей общей длины
                   1464: перегородок и в целом экономичнее, нежели другие фигуры. Пчелы, формируя
                   1465: соты, придерживаются как раз гексагональной структуры, которой и
                   1466: воспользовался архитектор. По другой версии - это дань уважения
                   1467: заказчику, на гербе которого изображены три пчелы.
                   1468: 
                   1469: Источник:
                   1470:    1. https://ru.wikipedia.org/wiki/Сант-Иво_алла_Сапиенца
                   1471:    2. Е. Джеро. Рим за три дня: Нескучный путеводитель.
                   1472: https://books.google.ru/books?id=DplGDwAAQBAJ&pg=PT119#v=onepage&q&f=false
                   1473:    3. https://theoryandpractice.ru/posts/15216-pchelinaya-ekonomiya-pochemu-priroda-predpochitaet-shestiugolniki
                   1474:    4. https://ru.wikipedia.org/wiki/Пчёлы-плотники
                   1475: 
                   1476: Автор:
                   1477: Евгений Ярков (Тюмень)
                   1478: 
                   1479: Вопрос 6:
                   1480: Для защиты от дурного слова при обращении к Марии Египетской нужно было
                   1481: погладить себя рукой по заду, а затем потереть той же рукой по лицу. В
                   1482: день памяти Марии могли быть предложены пустые щи. Назовите день и месяц
                   1483: по старому стилю, когда церковь чтила память Марии Египетской.
                   1484: 
                   1485: Ответ:
                   1486: 1 апреля.
                   1487: 
                   1488: Комментарий:
                   1489: В каждой шутке есть доля правды. В апреле у крестьян заканчивалась
                   1490: капуста, и они хлебали практически одну водицу. Действия суеверных
                   1491: людей, поминающих Марию, похожи на первоапрельский розыгрыш.
                   1492: 
                   1493: Источник:
                   1494: http://www.calend.ru/narodevent/6629/
                   1495: 
                   1496: Автор:
                   1497: Евгений Ярков (Тюмень)
                   1498: 
                   1499: Вопрос 7:
                   1500: Артемий Лебедев, узнав о том, что конкуренты подали в суд иск,
                   1501: касающийся разработки его студией символики Санкт-Петербурга, призвал не
                   1502: ДЕЛАТЬ ЭТО на логотип. Когда ЭТО ДЕЛАЕТ на голубей лирический герой
                   1503: песни Веры Абрамовой, птицы радуются. Ответьте двумя словами: что
                   1504: делает?
                   1505: 
                   1506: Ответ:
                   1507: Крошит батон.
                   1508: 
                   1509: Комментарий:
                   1510: "Крошить батон" означает "обвинять несправедливо", "наезжать не по
                   1511: делу". В Питере, как известно, батоны, а не булки. А голуби крошкам
                   1512: радуются.
                   1513:    z-checkdb: Всё ровно наоборот: в Питере именно булки, а батоны - в
                   1514: Москве, см. http://www.aif.ru/archive/1714627 (Сергей Сыромятников).
                   1515: 
                   1516: Источник:
                   1517:    1. https://tema.livejournal.com/2349059.html
                   1518:    2. http://music.lib.ru/a/abramowa_w_n/alb15.shtml
                   1519:    3. http://aav.ru/mospit/
                   1520:    4. https://dic.academic.ru/dic.nsf/proverbs/11633/
                   1521: 
                   1522: Автор:
                   1523: Евгений Ярков (Тюмень)
                   1524: 
                   1525: Вопрос 8:
                   1526: Знаменитый с античности город Бат англичане иногда называли "городом
                   1527: больных". Тем не менее, для обозначения подобных мест англичане в наши
                   1528: дни упоминают другой трехбуквенный город. Назовите этот другой город.
                   1529: 
                   1530: Ответ:
                   1531: Спа.
                   1532: 
                   1533: Комментарий:
                   1534: Бат - это бальнеологический курорт.
                   1535: 
                   1536: Источник:
                   1537:    1. https://ru.wikipedia.org/wiki/Бат_(Англия)
                   1538:    2. https://ru.wikipedia.org/wiki/Спа_(физиотерапия)
                   1539: 
                   1540: Автор:
                   1541: Евгений Ярков (Тюмень)
                   1542: 
                   1543: Вопрос 9:
                   1544: Кирилл Еськов замечает, что Толкин не описывал товарные отношения во
                   1545: "Властелине Колец". В зависимости от того, как героям приходилось решать
                   1546: финансовые проблемы - доблестью или хитростью, - Еськов предлагает две
                   1547: версии: считать эпопею ПЕРВЫМ или ВТОРЫМ. Назовите или ПЕРВЫЙ двумя
                   1548: словами на одну букву, или ВТОРОЙ - словами на соседние буквы алфавита.
                   1549: 
                   1550: Ответ:
                   1551: Рыцарский роман.
                   1552: 
                   1553: Зачет:
                   1554: Плутовской роман.
                   1555: 
                   1556: Комментарий:
                   1557: Если там герои вели себя доблестно и рассчитывались деньгами, не
                   1558: совершая обмана в денежных операциях, то это рыцарский роман. Если же
                   1559: денежные операции, так или иначе, были завязаны на хитрость - то
                   1560: плутовской, а может и криминальный.
                   1561: 
                   1562: Источник:
                   1563:    1. К.Ю. Еськов. Как и зачем я писал апокриф к "Властелину колец".
                   1564: http://flibusta.is/b/15917/read
                   1565:    2. https://ru.wikipedia.org/wiki/Рыцарский_роман
                   1566:    3. https://ru.wikipedia.org/wiki/Плутовской_роман
                   1567: 
                   1568: Автор:
                   1569: Евгений Ярков (Тюмень)
                   1570: 
                   1571: Тур:
                   1572: 4.2. Команда Евгения Волконского
                   1573: 
                   1574: Дата:
                   1575: 24-Feb-2018
                   1576: 
                   1577: Инфо:
                   1578: Команда горячо благодарит Серафима Шибанова и Александра Коробейникова
                   1579: за тестирование пакета и ряд ценных замечаний и предложений.
                   1580: 
                   1581: Вопрос 1:
                   1582: Созданный в 1853 году портрет этого "директора" не был писан с натуры,
                   1583: однако никто из исследователей ни разу не подверг сомнению его сходство
                   1584: с оригиналом. Чей же это был портрет?
                   1585: 
                   1586: Ответ:
                   1587: Козьмы Пруткова.
                   1588: 
                   1589: Комментарий:
                   1590: Как известно, Козьма Прутков дослужился до директора Пробирной Палатки.
                   1591: Портрет создали Александр Бейдеман, Лев Жемчужников и Лев Лагорио.
                   1592: Подвергнуть сомнению сходство с оригиналом было трудно за отсутствием
                   1593: последнего.
                   1594: 
                   1595: Источник:
                   1596: http://www.kozma.ru/directions/portrait.htm
                   1597: 
                   1598: Автор:
                   1599: Юлия Петрова, Михаил Розенблюм
                   1600: 
                   1601: Вопрос 2:
                   1602: Явление в английском языке, при котором существительное выходит за
                   1603: границы придаточного определительного предложения и уводит за собой
                   1604: предлог, называется термином, производным от НЕГО. Назовите ЕГО двумя
                   1605: словами, начинающимися на парные согласные.
                   1606: 
                   1607: Ответ:
                   1608: Гамельнский крысолов.
                   1609: 
                   1610: Комментарий:
                   1611: Автор надеется, что текущий вопрос станет у него единственным с этим
                   1612: ответом, потому что сколько ж можно, но удержаться он не смог.
                   1613: 
                   1614: Источник:
                   1615: https://lingthusiasm.com/post/167572233831/lingthusiasm-episode-14-getting-into-up-for-and
                   1616: 
                   1617: Автор:
                   1618: Игорь Мокин
                   1619: 
                   1620: Вопрос 3:
                   1621: Согласно славянской мифологии, ЕГО можно было выменять у нечистой силы
                   1622: за черную кошку или жаренного в перьях гусака. Если после совершения
                   1623: такой сделки вернуться домой не оглядываясь и ни с кем не разговаривая,
                   1624: то в этом доме до конца жизни будет достаток и богатство. Википедия
                   1625: пишет, что братья Стругацкие внесли в предание о НЕМ... Что именно?
                   1626: 
                   1627: Ответ:
                   1628: [Свои] пять копеек.
                   1629: 
                   1630: Зачет:
                   1631: По упоминанию пяти копеек или пятака.
                   1632: 
                   1633: Комментарий:
                   1634: ОН - это неразменный рубль. У Стругацких в повести "Понедельник
                   1635: начинается в субботу" фигурирует неразменный пятак.
                   1636: 
                   1637: Источник:
                   1638:    1. https://ru.wikipedia.org/wiki/Неразменный_рубль
                   1639:    2. https://ru.wikipedia.org/wiki/Проект:Знаете_ли_вы/Архив_рубрики/2018-02
                   1640: 
                   1641: Автор:
                   1642: Евгений Волконский
                   1643: 
                   1644: Вопрос 4:
                   1645: Джордж Хэдли выступал за сборную Вест-Индии по крИкету. Он был
                   1646: темнокожим, а в этой команде по негласному правилу капитаном мог быть
                   1647: только белый. Однажды Хэдли всё же дали побыть капитаном, а после его
                   1648: уже не назначали. Возможно, это было в декабре. С чем сравнила это
                   1649: краткое действо историк Хиллари Беклс?
                   1650: 
                   1651: Ответ:
                   1652: С сатурналиями.
                   1653: 
                   1654: Комментарий:
                   1655: Когда он на короткое время занял место "для белых", это было похоже на
                   1656: римский праздник Сатурналий, когда рабы на день занимали место господ.
                   1657: 
                   1658: Источник:
                   1659: Gideon Haigh. Game for Anything: Writings on Cricket.
                   1660: https://books.google.ru/books?id=LLREUs9Fjx4C&pg=PA36#v=onepage&q&f=false
                   1661: 
                   1662: Автор:
                   1663: Игорь Мокин
                   1664: 
                   1665: Вопрос 5:
                   1666: (pic: 20170788.jpg)
                   1667:    Напишите четырехбуквенное название этой картины Жузе Мальоа.
                   1668: 
                   1669: Ответ:
                   1670: "Фаду".
                   1671: 
                   1672: Источник:
                   1673: http://en.wikipedia.org/wiki/File:Jose_malhoa_fado.jpg
                   1674: 
                   1675: Автор:
                   1676: Игорь Мокин
                   1677: 
                   1678: Вопрос 6:
                   1679: Математик Юрий Манин известен также и своим умением красиво одеваться.
                   1680: Поэтому участники его семинара не слишком удивились, услышав от него в
                   1681: середине 1980-х фразу "Теперь появился неплохой шанс доказать
                   1682: [ПРОПУСК]". Заполните пропуск.
                   1683: 
                   1684: Ответ:
                   1685: ФирмУ; теорему фирмУ.
                   1686: 
                   1687: Зачет:
                   1688: Теорему фирмЫ (хоть так сказать нельзя, но различие слишком тонкое). С
                   1689: буквой "е" вместо "и" (фермУ) засчитывать, только если явно указано
                   1690: ударение на последний слог.
                   1691: 
                   1692: Комментарий:
                   1693: Сленговое выражение "фирмА" в позднесоветские годы означало
                   1694: высококачественный товар от зарекомендовавшего себя бренда. В середине
                   1695: 1980-х было доказано, что великая теорема Ферма следует из другой
                   1696: знаменитой гипотезы, которую позже доказал Эндрю Уайлз.
                   1697: 
                   1698: Источник:
                   1699: ЛОАВ.
                   1700: 
                   1701: Автор:
                   1702: Михаил Розенблюм
                   1703: 
                   1704: Вопрос 7:
                   1705: В одной сцене сериала "Сорвиголова" показан арест всех отрицательных
                   1706: героев, и мы понимаем, что ни один преступник не может спать спокойно.
                   1707: Силы закона одержат победу благодаря главному герою, который сохраняет
                   1708: инкогнито. Назовите композитора, произведение которого звучит в этой
                   1709: сцене за кадром.
                   1710: 
                   1711: Ответ:
                   1712: [Джакомо] Пуччини.
                   1713: 
                   1714: Комментарий:
                   1715: Драматургия описанной сцены, как и текст вопроса, представляет собой
                   1716: аллюзию на текст арии "Nessun dorma" принца Калафа из оперы "Турандот".
                   1717: 
                   1718: Источник:
                   1719:    1. Телесериал "Daredevil", s01e13.
                   1720:    2. https://en.wikipedia.org/wiki/Turandot
                   1721: 
                   1722: Автор:
                   1723: Игорь Мокин
                   1724: 
                   1725: Вопрос 8:
                   1726: В романе Александра Солженицына "В круге первом" описывается визит
                   1727: госпожи Рузвельт в Бутырскую тюрьму. Для показа ей тюремщики специально
                   1728: подготовили одну из камер, поменяв там условия содержания. По окончании
                   1729: визита при обыске обнаружилось, что один из заключенных нанес ущерб
                   1730: книге, временно оказавшейся в его распоряжении. За это он был подвергнут
                   1731: физическому наказанию в два этапа. Опишите оба этапа в правильном
                   1732: порядке.
                   1733: 
                   1734: Ответ:
                   1735: Его ударили сначала в правую щеку, а потом в левую.
                   1736: 
                   1737: Зачет:
                   1738: По смыслу.
                   1739: 
                   1740: Комментарий:
                   1741: Заключенный вырвал из принесенного в камеру Евангелия "Нагорную
                   1742: проповедь".
                   1743: 
                   1744: Источник:
                   1745:    1. А.И. Солженицын. В круге первом.
                   1746: https://books.google.ru/books?id=dSqiAAAAQBAJ&pg=RA2-PT438#v=onepage&q&f=false
                   1747:    2. https://ru.wikisource.org/wiki/От_Матфея_святое_благовествование#Глава_5
                   1748: 
                   1749: Автор:
                   1750: Михаил Розенблюм
                   1751: 
                   1752: Вопрос 9:
                   1753: В 1815 году по решению мятежного национального конгресса ОН был окружен
                   1754: ружьями, пушками и пушечными ядрами. В одном из вариантов 1821 года ОН
                   1755: стал смотреть вправо, однако при этом у НЕГО отобрали добычу. В 1823
                   1756: году добыча была возвращена на место. А в 1893 году ОН стал смотреть
                   1757: влево. Назовите ЕГО как можно точнее.
                   1758: 
                   1759: Ответ:
                   1760: Орел, пожирающий змею.
                   1761: 
                   1762: Зачет:
                   1763: По смыслу: например, орел на кактусе, мексиканский гербовый орел, орел
                   1764: со змеей и т.п.
                   1765: 
                   1766: Комментарий:
                   1767: 24 февраля в Мексике отмечается день национального флага.
                   1768: 
                   1769: Источник:
                   1770:    1. http://www.latintour.ru/meksika/mx-info/flag-mex.html
                   1771:    2. https://ru.wikipedia.org/wiki/Флаг_Мексики
                   1772: 
                   1773: Автор:
                   1774: Юрий Попов
                   1775: 
                   1776: Тур:
                   1777: 5.1. "Улыбаемся и пашем"
                   1778: 
                   1779: Дата:
                   1780: 24-Feb-2018
                   1781: 
                   1782: Редактор:
                   1783: Серафим Шибанов (Москва)
                   1784: 
                   1785: Вопрос 1:
                   1786: Согласно шутке, известный писатель показал, что, чтобы развалить
                   1787: социализм, достаточно трех ослов, а чтобы развалить капитализм,
                   1788: необходимо вторжение инопланетян. Назовите этого писателя.
                   1789: 
                   1790: Ответ:
                   1791: [Николай] Носов.
                   1792: 
                   1793: Комментарий:
                   1794: Коммунистическую утопию, описанную в книге "Незнайка в Солнечном
                   1795: городе", чуть было не разрушают трое ослов, которых Незнайка превратил в
                   1796: людей, а в книге "Незнайка на Луне" герои прилетают на Луну и успешно
                   1797: борются с тамошним капитализмом.
                   1798: 
                   1799: Источник:
                   1800: https://denis-strebkov.livejournal.com/241745.html?thread=2227793#t2227793
                   1801: 
                   1802: Автор:
                   1803: Лариса Шемтова
                   1804: 
                   1805: Вопрос 2:
                   1806: КаролИнский жнец, Нага ДжолОкия и Дыхание дракона - это всё ОНИ. ОНИ
                   1807: упоминаются в названии музыкальной группы. Какой именно?
                   1808: 
                   1809: Ответ:
                   1810: "Red Hot Chili Peppers" [чтецу: ред хот чИли пЕпперз].
                   1811: 
                   1812: Комментарий:
                   1813: ОНИ - это красные перцы чили.
                   1814: 
                   1815: Источник:
                   1816:    1. https://en.wikipedia.org/wiki/Carolina_Reaper
                   1817:    2. https://en.wikipedia.org/wiki/Bhut_jolokia
                   1818:    3. https://en.wikipedia.org/wiki/Dragon%27s_Breath_(chili_pepper)
                   1819:    4. https://en.wikipedia.org/wiki/Red_Hot_Chili_Peppers
                   1820: 
                   1821: Автор:
                   1822: Владимир Тихонов, в редакции Серафима Шибанова
                   1823: 
                   1824: Вопрос 3:
                   1825: Майкл ДеНиро и Сэмюель Эпштейн изучали факторы формирования изотопного
                   1826: состава животных и по итогам исследования опубликовали работу "[ПЯТЬ
                   1827: СЛОВ ПРОПУЩЕНЫ] (плюс несколько промилле)". Какие слова мы пропустили в
                   1828: тексте вопроса?
                   1829: 
                   1830: Ответ:
                   1831: Ты - то, что ты ешь.
                   1832: 
                   1833: Комментарий:
                   1834: Вариации в изотопном составе обычно невелики и выражаются в промилле.
                   1835: 
                   1836: Источник:
                   1837:    1. DeNiro M.J. and Epstein S. (1976) You are what you eat (plus a few
                   1838: permil): the carbon isotope cycle in food chains. Geological Society of
                   1839: America Abstracts with Programs 8: 834-835.
                   1840:    2. Brian Fry. Stable Isotope Ecology.
                   1841: https://books.google.ru/books?id=9O0Zsy48-hsC&pg=PA67#v=onepage&q&f=false
                   1842: 
                   1843: Автор:
                   1844: Лев Поздняков
                   1845: 
                   1846: Вопрос 4:
                   1847: (pic: 20170789.jpg)
                   1848:    Перед вами галочки-шевроны на скоростном шоссе на Кипре. Такие
                   1849: галочки расположены в одном ряду на протяжении нескольких километров.
                   1850: Что с помощью этих галочек рекомендуется делать водителю?
                   1851: 
                   1852: Ответ:
                   1853: Соблюдать дистанцию до впереди идущего автомобиля.
                   1854: 
                   1855: Зачет:
                   1856: Соблюдать дистанцию в размере двух шевронов; аналогичные ответы по
                   1857: смыслу.
                   1858: 
                   1859: Комментарий:
                   1860: (pic: 20170790.jpg)
                   1861: 
                   1862: Источник:
                   1863: Фото автора вопроса.
                   1864: 
                   1865: Автор:
                   1866: Сергей Паников
                   1867: 
                   1868: Вопрос 5:
                   1869: (pic: 20170791.jpg)
                   1870:    Недавно в Германии на автобане между Мюнхеном и Берлином появился
                   1871: новый дорожный знак, представленный на раздаточном материале. Для какой
                   1872: категории транспортных средств он предназначен?
                   1873: 
                   1874: Ответ:
                   1875: Для беспилотников.
                   1876: 
                   1877: Комментарий:
                   1878: Данный знак необходим исключительно для беспилотных автомобилей, для
                   1879: корректировки их положения на дороге.
                   1880: 
                   1881: Источник:
                   1882: https://liveberlin.ru/news/2017/02/14/autonomes-fahren-verkehrsschilder/
                   1883: 
                   1884: Автор:
                   1885: Владимир Тихонов
                   1886: 
                   1887: Вопрос 6:
                   1888: Помпей Великий говорил своим сторонникам, что стоит ему только СДЕЛАТЬ
                   1889: ЭТО в любом месте Италии, как тотчас же появится его войско. СДЕЛАТЬ ЭТО
                   1890: с подобным же эффектом грозился и заглавный персонаж известного
                   1891: произведения первой четверти XX века. Назовите это произведение.
                   1892: 
                   1893: Ответ:
                   1894: "Мойдодыр".
                   1895: 
                   1896: Комментарий:
                   1897: И тот, и другой хвалились, что им достаточно топнуть ногой - и тотчас же
                   1898: явится их армия.
                   1899: 
                   1900: Источник:
                   1901:    1. Плутарх. Сравнительные жизнеописания в двух томах. Помпей.
                   1902: http://ancientrome.ru/antlitr/t.htm?a=1439003200#57
                   1903:    2. К.И. Чуковский. Мойдодыр.
                   1904: http://www.chukfamily.ru/kornei/tales/mojdodyr
                   1905: 
                   1906: Автор:
                   1907: Лев Поздняков
                   1908: 
                   1909: Вопрос 7:
                   1910: Персонаж Дмитрия Гайдука Гена Психоделик однажды понял, что у кошки не
                   1911: только снаружи усы, у нее еще и внутри усы. Развивая свою мысль, герой
                   1912: вспоминает ЕЕ. Назовите ЕЕ двумя словами, начинающимися на соседние
                   1913: буквы алфавита.
                   1914: 
                   1915: Ответ:
                   1916: Теория суперструн.
                   1917: 
                   1918: Зачет:
                   1919: Теория струн.
                   1920: 
                   1921: Комментарий:
                   1922: "И вся вселенная тоже состоит из усов, а вернее, из голографических
                   1923: суперструн - об этом, кажется, кто-то уже писал: наворотил каких-то
                   1924: стрёмных терминов...".
                   1925: 
                   1926: Источник:
                   1927: Д. Гайдук. Про одинаковых людей.
                   1928: http://www.rastamantales.com/skazki/content/pro-odinakovyh-lyudey
                   1929: 
                   1930: Автор:
                   1931: Лев Поздняков
                   1932: 
                   1933: Вопрос 8:
                   1934: Когда железные пароходы вытеснили деревянные баржи, работу потеряли не
                   1935: только бурлаки, но и другие профессионалы. И если бурлаки так и остались
                   1936: не у дел, то профессионалы нашли новый объект для своего искусства. В
                   1937: каком городе находится самый прославленный из этих объектов?
                   1938: 
                   1939: Ответ:
                   1940: Вологда.
                   1941: 
                   1942: Комментарий:
                   1943: С XVI века волжские расшивы было принято украшать затейливой резьбой.
                   1944: Считалось, что она помогает кораблю подняться против течения. Когда в
                   1945: 1870-е годы пароходы вытеснили с реки деревянные барки, мастера
                   1946: разбрелись в поисках заработка, и в деревянном зодчестве Средней России
                   1947: наступила 30-летняя эпоха великолепных резных наличников и других резных
                   1948: деталей. Благодаря "Песнярам" всем известно, что в Вологде есть дом с
                   1949: резным палисадом.
                   1950: 
                   1951: Источник:
                   1952: http://www.vokrugsveta.ru/vs/article/7776/
                   1953: 
                   1954: Автор:
                   1955: Мария Дуброва
                   1956: 
                   1957: Вопрос 9:
                   1958: Внимание, черный ящик!
                   1959:    Рассказывают, что однажды Марк Твен провел полчаса в гостях у
                   1960: знакомой. По возвращении жена Твена обратила внимание на одно упущение и
                   1961: стала упрекать мужа. Тогда Твен отправил подруге маленький сверток с НИМ
                   1962: и записку с просьбой посмотреть на НЕГО полчаса, после чего вернуть
                   1963: обратно - ведь ОН у Марка Твена единственный. Назовите ЕГО.
                   1964: 
                   1965: Ответ:
                   1966: Галстук.
                   1967: 
                   1968: Комментарий:
                   1969: В те времена появляться в обществе без галстука или вовсе с открытой
                   1970: шеей считалось неприличным. Марк Твен решил, что если его подруга, а это
                   1971: была Гарриет Бичер-Стоу, полчаса смотрела на него без галстука - то
                   1972: получасовое лицезрение галстука без него сгладит ситуацию.
                   1973: 
                   1974: Источник:
                   1975: https://www.nkj.ru/archive/articles/6096/
                   1976: 
                   1977: Автор:
                   1978: Лев Поздняков
                   1979: 
                   1980: Тур:
                   1981: 5.2. "Трамвай Еиналеж"
                   1982: 
                   1983: Дата:
                   1984: 24-Feb-2018
                   1985: 
                   1986: Редактор:
                   1987: Серафим Шибанов (Москва)
                   1988: 
                   1989: Инфо:
                   1990: Команда благодарит Андрея Гусева (Рига), Александра Ефименко, Давида
                   1991: Кадагидзе (оба - Москва), Ирину Медноногову (Санкт-Петербург) и команды
                   1992: "Банк Восточный" (Москва), "Ливония" (Таллин) и "Черурака" (Москва) за
                   1993: тестирование вопросов и ценные советы.
                   1994: 
                   1995: Вопрос 1:
                   1996: (pic: 20170792.jpg)
                   1997:    Новички чемпионата Москвы команда "Трамвай Еиналеж" приветствуют
                   1998: соперников!
                   1999:    В первый год выступлений в "Формуле-1" двигатели команды "Рено"
                   2000: отличались низкой надежностью. В 2017 году в честь юбилея дебюта
                   2001: компания "Рено" выпустила сувенир - окрашенный в фирменный желтый цвет
                   2002: предмет домашнего обихода. Какой именно?
                   2003: 
                   2004: Ответ:
                   2005: [Заварочный] чайник.
                   2006: 
                   2007: Комментарий:
                   2008: Моторы болидов часто горели, испуская клубы дыма. За это соперники
                   2009: окрестили машины "Рено" желтыми чайниками. Еще чайниками обычно именуют
                   2010: новичков.
                   2011: 
                   2012: Источник:
                   2013: https://www.wired.com/story/renault-f1-yellow-teapot/
                   2014: 
                   2015: Автор:
                   2016: Михаил Волхонский
                   2017: 
                   2018: Вопрос 2:
                   2019: [Ведущему: В слове "обоРЗение" опечатки нет! О заглавных буквах в словах
                   2020: "Крабов" и "Кальмаров" не упоминать; двоеточие не озвучивать!]
                   2021:    В юмористическом рассказе "Спортивное оборзение" несколько раз
                   2022: упоминаются участники матча: Крабов и Кальмаров. Матча по какому виду
                   2023: спорта?
                   2024: 
                   2025: Ответ:
                   2026: Шахматы.
                   2027: 
                   2028: Комментарий:
                   2029: Крабов и Кальмаров в данном случае - фамилии. Это аллюзия на длительные
                   2030: матчи Карпова и Каспарова за звание чемпиона мира.
                   2031: 
                   2032: Источник:
                   2033: К.С. Мелихан. Спортивное оборзение. http://flibusta.is/b/98496/read#t125
                   2034: 
                   2035: Автор:
                   2036: Сергей Ошанин
                   2037: 
                   2038: Вопрос 3:
                   2039:    <раздатка>
                   2040:    Giannis Antetokounmpo
                   2041:    </раздатка>
                   2042:    Греческий баскетболист Яннис АдетокУнбо получил в НБА прозвище "ИКС".
                   2043: Слово "ИКС" тоже имеет греческое происхождение. На самом деле прозвище
                   2044: справедливо только на 46%. Назовите ИКС.
                   2045: 
                   2046: Ответ:
                   2047: Алфавит.
                   2048: 
                   2049: Комментарий:
                   2050: Яннис получил прозвище "Алфавит" за обилие букв в имени и фамилии,
                   2051: однако его полное имя содержит всего 12 из 26 букв английского алфавита.
                   2052: 
                   2053: Источник:
                   2054:    1. https://www.basketball-reference.com/players/a/antetgi01.html
                   2055:    2. https://www.brewhoop.com/2013/10/17/4847356/giannis-antetokounmpo-nickname
                   2056: 
                   2057: Автор:
                   2058: Александр Ефименко
                   2059: 
                   2060: Вопрос 4:
                   2061:    <раздатка>
                   2062:    Japage 3
                   2063:    </раздатка>
                   2064:    Один коллектив на раннем этапе своего существования носил название
                   2065: Japage 3. Позже к этому коллективу присоединился человек, известный под
                   2066: псевдонимом. Под каким?
                   2067: 
                   2068: Ответ:
                   2069: Ринго Старр.
                   2070: 
                   2071: Комментарий:
                   2072: Тройка в названии говорит о том, что участников на тот момент было трое,
                   2073: а слово "Japage" содержит первые буквы имен Джон, Пол и Джордж. Позже
                   2074: эта группа превратилась в "The Beatles".
                   2075: 
                   2076: Источник:
                   2077: https://www.theguardian.com/books/2013/oct/02/beatles-tune-in-mark-lewisohn-review
                   2078: 
                   2079: Автор:
                   2080: Михаил Волхонский
                   2081: 
                   2082: Вопрос 5:
                   2083: У живущего во враждебном окружении курдского народа есть пословица: "У
                   2084: курдов нет друзей, кроме [ПРОПУСК 1]". Главный шанс в своей жизни
                   2085: [ПРОПУСК 2] упустил в 2000 году. Заполните любой из пропусков, если на
                   2086: слух они идентичны.
                   2087: 
                   2088: Ответ:
                   2089: Гор.
                   2090: 
                   2091: Комментарий:
                   2092: Курды доверяют только горам родного Курдистана. Альберт Гор в 2000 году
                   2093: проиграл президентские выборы Джорджу Бушу-младшему.
                   2094: 
                   2095: Источник:
                   2096:    1. https://www.inopressa.ru/article/26Sep2017/wp/kurds_israel.html
                   2097:    2. https://ru.wikipedia.org/wiki/Курдистан
                   2098:    3. https://ru.wikipedia.org/wiki/Гор,_Альберт
                   2099: 
                   2100: Автор:
                   2101: Александр Ефименко
                   2102: 
                   2103: Вопрос 6:
                   2104: Говоря об образе известного персонажа, кинокритик Боб Чипман отмечает,
                   2105: что цирковые акробаты и силачи в первой половине прошлого века из-за
                   2106: несовершенства текстиля вынуждены были ДЕЛАТЬ ЭТО. Какие четыре слова мы
                   2107: заменили словами "ДЕЛАТЬ ЭТО"?
                   2108: 
                   2109: Ответ:
                   2110: Надевать трусы поверх трико.
                   2111: 
                   2112: Зачет:
                   2113: Синонимичные ответы с сохранением формы (например, плавки/шорты вместо
                   2114: трусов, костюм/форма/штаны/брюки/треники/кальсоны вместо трико и т.п.).
                   2115: 
                   2116: Комментарий:
                   2117: Чтобы избежать конфуза, если трико лопнет, артисты цирка надевали поверх
                   2118: него трусы. Их внешний вид отразился в образе Супермена и других
                   2119: супергероев. Кстати, в новых фильмах про Супермена эта деталь
                   2120: отсутствует, что раздражает Чипмана.
                   2121: 
                   2122: Источник:
                   2123: https://www.youtube.com/watch?v=F9juReoJxI0&t=25m16s
                   2124: 
                   2125: Автор:
                   2126: Михаил Волхонский
                   2127: 
                   2128: Вопрос 7:
                   2129: В скороговорке литературоведа Михаила Безродного одному ницшеанцу
                   2130: повсюду мерещится ОНА. Назовите ЕЕ одним словом.
                   2131: 
                   2132: Ответ:
                   2133: Процентщица.
                   2134: 
                   2135: Комментарий:
                   2136: Идеям Ницше во многом созвучны идеи Родиона Раскольникова. Для того
                   2137: чтобы фраза осталась скороговоркой, в пропущенном слове должно быть
                   2138: много шипящих и свистящих звуков.
                   2139: 
                   2140: Источник:
                   2141:    1. М.В. Безродный. Конец цитаты. - СПб.: Издательство Ивана Лимбаха,
                   2142: 1996. - С. 153
                   2143: (http://imwerden.de/pdf/bezrodny_konets_tsytaty_1996_text.pdf).
                   2144:    2. http://www.nietzsche.ru/around/russia/jeits/
                   2145: 
                   2146: Автор:
                   2147: Михаил Волхонский
                   2148: 
                   2149: Вопрос 8:
                   2150: В юмористическом рассказе Константина Мелихана два брата последовательно
                   2151: теряли под диваном шашку, бочонок лото и рюху от городков, но каждый раз
                   2152: им удавалось заменить потерянный предмет. В итоге их дедушка перестал
                   2153: пользоваться ЕЮ. Назовите ЕЕ односложным словом.
                   2154: 
                   2155: Ответ:
                   2156: Трость.
                   2157: 
                   2158: Комментарий:
                   2159: Замену потерянным предметам братья отпиливали от дедушкиной трости. В
                   2160: результате дедушка больше не мог на нее опираться и перестал хромать.
                   2161: 
                   2162: Источник:
                   2163: К.С. Мелихан. Палочка-выручалочка. http://flibusta.is/b/98496/read#t130
                   2164: 
                   2165: Автор:
                   2166: Сергей Ошанин
                   2167: 
                   2168: Вопрос 9:
                   2169: Англоязычные преподаватели иногда рекомендуют своим ученикам запомнить
                   2170: пару фраз: "Эдди съел динамит. До свидания, Эдди". Ответьте точно:
                   2171: преподаватели чего?
                   2172: 
                   2173: Ответ:
                   2174: Игры на шестиструнной гитаре.
                   2175: 
                   2176: Зачет:
                   2177: По упоминанию гитары.
                   2178: 
                   2179: Комментарий:
                   2180: Первые буквы слов "Eddie Ate Dynamite, Good Bye Eddie" соответствуют
                   2181: шести струнам гитары - ми-ля-ре-соль-си-ми. Это наш последний вопрос. Мы
                   2182: прощаемся не только с Эдди, но и с вами. Спасибо за игру!
                   2183: 
                   2184: Источник:
                   2185: https://www.theguitarlesson.com/guitar-lesson-blog/beginner-guitar-lessons/guitar-string-mnemonic-acronyms/
                   2186: 
                   2187: Автор:
                   2188: Михаил Волхонский
                   2189: 
                   2190: Тур:
                   2191: 6.1. "Выдра в гетрах"
                   2192: 
                   2193: Дата:
                   2194: 24-Feb-2018
                   2195: 
                   2196: Редактор:
                   2197: Серафим Шибанов (Москва)
                   2198: 
                   2199: Инфо:
                   2200: Редактор благодарит за помощь в тестировании вопросов команду "Проект
                   2201: Церебро" (Астрахань).
                   2202: 
                   2203: Вопрос 1:
                   2204: Сооснователь сети ресторанов в интервью указал, что в ресторан нанимали
                   2205: и нанимают поваров-эмпатов, так как те обладают экспертными знаниями и
                   2206: навыками в области национальной кухни. В одном из слов предыдущего
                   2207: предложения мы заменили две буквы одной. Напишите это слово в исходном
                   2208: виде.
                   2209: 
                   2210: Ответ:
                   2211: Экспатов.
                   2212: 
                   2213: Комментарий:
                   2214: По словам одного из основателей, в сеть ресторанов предпочитают нанимать
                   2215: поваров-экспатов.
                   2216: 
                   2217: Источник:
                   2218: http://blog.elementaree.ru/heroes/ruslan-radzhapov
                   2219: 
                   2220: Автор:
                   2221: Виталий Литвинов
                   2222: 
                   2223: Вопрос 2:
                   2224: В современном отечественном романе основную часть ЕЕ населения
                   2225: составляют бывшие комсомольские деятели, выжившие ветераны и партийные
                   2226: функционеры. Напишите ЕЕ официальное название.
                   2227: 
                   2228: Ответ:
                   2229: Сокольническая линия.
                   2230: 
                   2231: Комментарий:
                   2232: В романе Дмитрия Глуховского "Метро 2033" на красной ветке метро жили, в
                   2233: основном, коммунисты, ветераны войны и партийные функционеры.
                   2234: 
                   2235: Источник:
                   2236:    1. http://ru.metro.wikia.com/wiki/Красная_Линия
                   2237:    2. https://ru.wikipedia.org/wiki/Метро_2033
                   2238:    3. https://ru.wikipedia.org/wiki/Сокольническая_линия
                   2239: 
                   2240: Автор:
                   2241: Сергей Бакшаев
                   2242: 
                   2243: Вопрос 3:
                   2244: Геракл сумел СДЕЛАТЬ ЭТО только в Гиперборее. Можно сказать, что испанцы
                   2245: не смогли СДЕЛАТЬ ЭТО у берегов Вальпараисо в 1580 году. Ответьте двумя
                   2246: словами: что такое "СДЕЛАТЬ ЭТО"?
                   2247: 
                   2248: Ответ:
                   2249: Догнать лань.
                   2250: 
                   2251: Зачет:
                   2252: Поймать лань; настичь лань.
                   2253: 
                   2254: Комментарий:
                   2255: Геракл очень долго гонялся за быстроногой керинейской ланью и сумел
                   2256: настичь ее только в северной Гиперборее. В 1580 году испанцы бросились в
                   2257: погоню за кораблем Фрэнсиса Дрейка "Золотая лань", который разорял
                   2258: берега Чили, но настичь его так и не смогли.
                   2259: 
                   2260: Источник:
                   2261:    1. https://ru.wikipedia.org/wiki/Керинейская_лань
                   2262:    2. И.А. Медведев. В поисках Эльдорадо.
                   2263: http://flibusta.is/b/417051/read
                   2264: 
                   2265: Автор:
                   2266: Сергей Бакшаев
                   2267: 
                   2268: Вопрос 4:
                   2269: В романе Уильяма Теккерея перед отправкой на войну юные прапорщики пишут
                   2270: родителям письма, "полные любви, сердечности, отваги и ИХ". Назовите ИХ
                   2271: двумя словами, начинающимися на одну и ту же букву.
                   2272: 
                   2273: Ответ:
                   2274: Орфографические ошибки.
                   2275: 
                   2276: Источник:
                   2277: У.М. Теккерей. Ярмарка тщеславия. http://flibusta.is/b/470374/read
                   2278: 
                   2279: Автор:
                   2280: Александра Башина
                   2281: 
                   2282: Вопрос 5:
                   2283: В Средние века [ПРОПУСК] был так плотно застроен домами и лавками, что
                   2284: невозможно было увидеть реку. Свое название этот [ПРОПУСК] неоднократно.
                   2285: Заполните любой из пропусков двумя словами.
                   2286: 
                   2287: Ответ:
                   2288: Мост Менял.
                   2289: 
                   2290: Зачет:
                   2291: Мост менял.
                   2292: 
                   2293: Комментарий:
                   2294: Мост Менял в Париже в свое время был очень плотно застроен домами и
                   2295: лавками торговцев и ремесленников. За свою историю этот мост менял свое
                   2296: название несколько раз.
                   2297: 
                   2298: Источник:
                   2299: https://ru.wikipedia.org/wiki/Мост_Менял
                   2300: 
                   2301: Автор:
                   2302: Сергей Бакшаев
                   2303: 
                   2304: Вопрос 6:
                   2305: В известном рассказе английской писательницы ОНИ затмевали небо и
                   2306: выстраивались в боевом порядке. "ОНА" была использована во время
                   2307: празднования в Москве 50-летнего юбилея в 2013 году. Назовите ЕЕ одним
                   2308: словом.
                   2309: 
                   2310: Ответ:
                   2311: Чайка.
                   2312: 
                   2313: Комментарий:
                   2314: В первом случае речь идет о рассказе Дафны Дюморье "Птицы". Во втором -
                   2315: о юбилее полета Валентины Терешковой, на праздновании которого выкатили
                   2316: автомобиль "Чайку", что символизировало позывной первой женщины в
                   2317: космосе.
                   2318: 
                   2319: Источник:
                   2320:    1. Д. Дю Морье. Птицы. http://flibusta.is/b/240282/read
                   2321:    2. https://ru.wikipedia.org/wiki/ГАЗ-13
                   2322: 
                   2323: Автор:
                   2324: Сергей Бакшаев
                   2325: 
                   2326: Вопрос 7:
                   2327: (pic: 20170793.jpg)
                   2328:    Перед вами ОНА. По запросу "ОНИ" Google выдает в числе прочих
                   2329: изображение ландтага. Назовите ЕЕ двумя словами.
                   2330: 
                   2331: Ответ:
                   2332: Фотография Лихтенштейна.
                   2333: 
                   2334: Комментарий:
                   2335: Вы можете увидеть перед собой фотографию Роя Лихтенштейна - известного
                   2336: представителя поп-арта. На фоне фотографии можно разглядеть характерные
                   2337: для творчества художника комиксы. Если запросить у интернет-поисковика
                   2338: фотографии Лихтенштейна, то на одной из них вы увидите местный
                   2339: парламент.
                   2340: 
                   2341: Источник:
                   2342:    1. http://fb.ru/article/231739/roy-lihtenshteyn---sozdatel-stilya-pop-art
                   2343:    2. https://www.google.ru/search?q=фотографии+лихтенштейна&tbm=isch
                   2344:    3. https://ru.wikipedia.org/wiki/Ландтаг_Лихтенштейна
                   2345: 
                   2346: Автор:
                   2347: ???
                   2348: 
                   2349: Вопрос 8:
                   2350: По мнению биографа Витторио Торторелли, ОН обладал волшебной силой и
                   2351: проникновенностью, согревал сердца и вызывал их трепет.
                   2352: Оториноларинголог Альфред Томатис, говоря о НЕМ, привел величину в 8000
                   2353: герц. Назовите ЕГО двумя словами, начинающимися на парные согласные.
                   2354: 
                   2355: Ответ:
                   2356: Голос Карузо.
                   2357: 
                   2358: Комментарий:
                   2359: Биограф Витторио Торторелли так отзывался о голосе Энрико Карузо.
                   2360: Известный врач Альфред Томатис выяснил, что диапазон голоса Карузо
                   2361: составлял 8000 герц.
                   2362: 
                   2363: Источник:
                   2364:    1. В. Торторелли. Энрико Карузо.
                   2365: http://www.classic-music.ru/caruso19.html
                   2366:    2. Н. Дойдж. Мозг, исцеляющий сам себя. Реальные истории людей,
                   2367: которые победили болезни.
                   2368: https://books.google.ru/books?id=1c00DwAAQBAJ&pg=PT400#v=onepage&q&f=false
                   2369: 
                   2370: Автор:
                   2371: Сергей Бакшаев
                   2372: 
                   2373: Вопрос 9:
                   2374: В произведении Владимира Набокова герой смотрит на каплю, упавшую на
                   2375: текст. Далее писатель упоминает ЭТО. ЭТО было названо в честь человека,
                   2376: умершего около 2000 лет назад. Назовите ЭТО.
                   2377: 
                   2378: Ответ:
                   2379: Cicero.
                   2380: 
                   2381: Зачет:
                   2382: Цицеро.
                   2383: 
                   2384: Комментарий:
                   2385: Капля выполнила функцию лупы, увеличив текст, поэтому первоначальный
                   2386: петит превратился в более крупный цицеро. Кегль цицеро назван в честь
                   2387: Марка Туллия Цицерона.
                   2388: 
                   2389: Источник:
                   2390:    1. В.В. Набоков. Приглашение на казнь.
                   2391: http://flibusta.is/b/322391/read
                   2392:    2. https://ru.wikipedia.org/wiki/Цицеро
                   2393:    3. https://ru.wikipedia.org/wiki/Марк_Туллий_Цицерон
                   2394: 
                   2395: Автор:
                   2396: Сергей Бакшаев
                   2397: 
                   2398: Тур:
                   2399: 6.2. "MUSA"
                   2400: 
                   2401: Дата:
                   2402: 24-Feb-2018
                   2403: 
                   2404: Редактор:
                   2405: Серафим Шибанов (Москва)
                   2406: 
                   2407: Вопрос 1:
                   2408: Алика Калайда, говоря о гробиках для сувенирных фигурок Дракулы, пишет,
                   2409: что те нужны, чтобы граф мог спать там в [ПРОПУСК]. Георгий Петраш
                   2410: предположил, что узор на северо-западной башне Спасо-Преображенского
                   2411: собора в Чернигове - это [ПРОПУСК]. Заполните любой из пропусков двумя
                   2412: словами.
                   2413: 
                   2414: Ответ:
                   2415: Солнечные часы.
                   2416: 
                   2417: Комментарий:
                   2418: Вампир Дракула должен прятаться от солнца, поэтому солнечные часы дня
                   2419: предпочитает проводить в гробу. Узор на башне черниговского собора,
                   2420: возможно, помогал определять время по тому, какие его части были
                   2421: освещены, а какие оставались в тени.
                   2422: 
                   2423: Источник:
                   2424:    1. https://rikki-t-tavi.livejournal.com/1955864.html
                   2425:    2. http://www.gorod.cn.ua/news_14901.html
                   2426: 
                   2427: Автор:
                   2428: Анна Бабушкина
                   2429: 
                   2430: Вопрос 2:
                   2431: Описывая выбор одного из двух возможных путей персонажами, доехавшими до
                   2432: развилки, Теофиль Готье упоминает ландшафтный ИКС. На форуме,
                   2433: посвященном танцам, для разработки природного ИГРЕКА советуют сидеть
                   2434: между двух стульев - сам на одном, а ноги на другом. Ответьте в
                   2435: правильном порядке: что мы заменили ИКСОМ и ИГРЕКОМ?
                   2436: 
                   2437: Ответ:
                   2438: Игрек, Икс.
                   2439: 
                   2440: Зачет:
                   2441: Y, X.
                   2442: 
                   2443: Комментарий:
                   2444: В романе Готье упоминалась развилка, где одна дорога разделялась на две,
                   2445: т.е. напоминала букву ИГРЕК. Природным ИКСОМ ног называется немаловажное
                   2446: в балете умение выгибать колени в противоположную сторону.
                   2447: 
                   2448: Источник:
                   2449:    1. Т. Готье. Мадемуазель де Мопен. http://flibusta.is/b/424727/read
                   2450:    2. https://vk.com/topic-11665068_25123267
                   2451: 
                   2452: Автор:
                   2453: Анна Бабушкина
                   2454: 
                   2455: Вопрос 3:
                   2456: Одна школьница, прочитав "Дубровского", заявила, что в этом произведении
                   2457: у нее сочувствие вызывают только [ПРОПУСК]. "[ПРОПУСК]" - название
                   2458: современного отечественного анимационного сериала. Заполните пропуск.
                   2459: 
                   2460: Ответ:
                   2461: Маша и медведь.
                   2462: 
                   2463: Источник:
                   2464:    1. ЛОАВ.
                   2465:    2. https://ru.wikipedia.org/wiki/Маша_и_Медведь
                   2466: 
                   2467: Автор:
                   2468: Анна Бабушкина
                   2469: 
                   2470: Вопрос 4:
                   2471: На одном шуточном рисунке Эйнштейн и Пифагор отнимают друг у друга...
                   2472: Что?
                   2473: 
                   2474: Ответ:
                   2475: C^2.
                   2476: 
                   2477: Источник:
                   2478: (pic: 20170794.jpg)
                   2479: 
                   2480: Автор:
                   2481: Юлия Луговская
                   2482: 
                   2483: Вопрос 5:
                   2484: Обсуждая неизбежность ранних подъемов, читатель одного из форумов
                   2485: рассказывал, что пережить стадии утреннего пробуждения ему помогают пять
                   2486: будильников. Этим будильникам он дал имена. Назовите последнее из них.
                   2487: 
                   2488: Ответ:
                   2489: Принятие.
                   2490: 
                   2491: Комментарий:
                   2492: Будильники назывались - отрицание, гнев, торг, депрессия, принятие.
                   2493: 
                   2494: Источник:
                   2495: https://twitter.com/Standup_Ural/status/817632021296062465/
                   2496: 
                   2497: Автор:
                   2498: Юлия Луговская
                   2499: 
                   2500: Вопрос 6:
                   2501: Теофиль Готье, рассуждая о театре, писал: "Среди этих пьес, которые
                   2502: пишутся для фей и разыгрываются при лунном свете, есть одна - она-то
                   2503: меня главным образом и пленяет; пьеса эта такая бродячая, такая кочевая,
                   2504: интрига в ней так туманна, а характеры так причудливы, что сам автор, не
                   2505: зная, как ее озаглавить, дал ей название "[ПРОПУСК]" - название
                   2506: растяжимое и подходящее к чему угодно". Заполните пропуск.
                   2507: 
                   2508: Ответ:
                   2509: "Как вам это понравится".
                   2510: 
                   2511: Источник:
                   2512: Т. Готье. Мадемуазель де Мопен. http://flibusta.is/b/424727/read
                   2513: 
                   2514: Автор:
                   2515: Анна Бабушкина
                   2516: 
                   2517: Вопрос 7:
                   2518: Копимизм - зародившееся в XXI веке в Швеции религиозное движение,
                   2519: главным священным принципом которого является свободный файлообмен. В
                   2520: России веру, в основе которой лежат главные принципы копимизма,
                   2521: исповедует РПЦ. Расшифруйте эту аббревиатуру.
                   2522: 
                   2523: Ответ:
                   2524: Русская пиратская церковь.
                   2525: 
                   2526: Зачет:
                   2527: Российская пиратская церковь.
                   2528: 
                   2529: Источник:
                   2530: https://ru.wikipedia.org/wiki/Миссионерская_церковь_копимизма
                   2531: 
                   2532: Автор:
                   2533: Анна Бабушкина
                   2534: 
                   2535: Вопрос 8:
                   2536: В одной статье о посуде иронически советовали не путать соусник и
                   2537: бурдалУ. БурдалУ названы так по фамилии иезуитского священника,
                   2538: проповеди которого якобы были чрезвычайно ТАКИМИ. Какими - ТАКИМИ?
                   2539: 
                   2540: Ответ:
                   2541: Длинными.
                   2542: 
                   2543: Зачет:
                   2544: Долгими, многословными и т.п. по смыслу.
                   2545: 
                   2546: Комментарий:
                   2547: БурдалУ - предмет обихода XVII-XVIII веков, служивший дамам в кринолинах
                   2548: днем для тех же целей, для которых ночью использовалась ночная ваза.
                   2549: Сосуд формой напоминает соусник. По легенде, он и происходит от
                   2550: соусника, примененного не по назначению во время невыносимо длинной
                   2551: проповеди Луи БурдалУ.
                   2552: 
                   2553: Источник:
                   2554:    1. https://www.liveinternet.ru/users/5153342/post349531846/
                   2555:    2. https://ru.wikipedia.org/wiki/Бурдалю
                   2556: 
                   2557: Автор:
                   2558: Анна Бабушкина
                   2559: 
                   2560: Вопрос 9:
                   2561: "Отрекаюсь от своей ошибки, забираю эту песню вспять", - написал
                   2562: известный человек в 1989 году, приехав на курорт в дождливый сезон.
                   2563: Напишите первую строку песни, которую этот человек имел в виду.
                   2564: 
                   2565: Ответ:
                   2566: "У природы нет плохой погоды".
                   2567: 
                   2568: Комментарий:
                   2569: Это был Эльдар Рязанов.
                   2570: 
                   2571: Источник:
                   2572: Э.А. Рязанов. На отдыхе. http://flibusta.is/b/230643/read#t120
                   2573: 
                   2574: Автор:
                   2575: Анна Бабушкина
                   2576: 
                   2577: Тур:
                   2578: 7.1.  "Venceremos"
                   2579: 
                   2580: Дата:
                   2581: 19-May-2018
                   2582: 
                   2583: Редактор:
                   2584: Григорий Смыслов (Зеленоград)
                   2585: 
                   2586: Инфо:
                   2587: Тестеры пакета: Дмитрий Осипков (Зеленоград), Юлия Кожевникова,
                   2588: Владислав Маркович, Артур Кадурин (все - Москва).
                   2589: 
                   2590: Вопрос 1:
                   2591: В 1890 году ЕГО при попытке ареста выстрелом в голову убил человек по
                   2592: фамилии Буллхед. Назовите ЕГО.
                   2593: 
                   2594: Ответ:
                   2595: Сидящий Бык.
                   2596: 
                   2597: Зачет:
                   2598: Ситтинг Булл.
                   2599: 
                   2600: Комментарий:
                   2601: Сидящий Бык - пожалуй, один из самых известных индейских вождей. Был
                   2602: застрелен во время индейского восстания в 1890 году при попытке оказать
                   2603: сопротивление при аресте. Довольно занятно, что Сидящего Быка (Ситтинг
                   2604: Булла) выстрелом в голову убил человек по фамилии Буллхед (что в
                   2605: буквальном переводе означает "Бычья Голова").
                   2606: 
                   2607: Источник:
                   2608: https://ru.wikipedia.org/wiki/Сидящий_Бык
                   2609: 
                   2610: Автор:
                   2611: Алексей Кудяков
                   2612: 
                   2613: Вопрос 2:
                   2614:    <раздатка>
                   2615:    Владимир Константинов
                   2616:    Вячеслав Фетисов
                   2617:    Сергей Фёдоров
                   2618:    Игорь Ларионов
                   2619:    Вячеслав Козлов
                   2620:    </раздатка>
                   2621:    Рассказывая о фантастической игре "русской пятерки", автор сайта
                   2622: sports.ru использует слегка измененное название известного произведения.
                   2623: Назовите это произведение.
                   2624: 
                   2625: Ответ:
                   2626: "[Удивительный] волшебник страны Оз".
                   2627: 
                   2628: Комментарий:
                   2629: Русские хоккеисты творили на льду настоящую магию, и автор статьи
                   2630: называет их "волшебниками из страны Ов", обыгрывая одновременно их
                   2631: фамилии и знаменитую сказку Фрэнка Баума.
                   2632: 
                   2633: Источник:
                   2634: https://www.sports.ru/tribuna/blogs/centre/1302000.html
                   2635: 
                   2636: Автор:
                   2637: Алексей Кудяков
                   2638: 
                   2639: Вопрос 3:
                   2640: В начале Второй мировой войны к НЕМУ в дом из Лондона были эвакуированы
                   2641: несколько детей. Для того чтобы хоть как-то их развлечь, ОН начал
                   2642: рассказывать им истории. Назовите ЕГО.
                   2643: 
                   2644: Ответ:
                   2645: [Клайв Стейплз] Льюис.
                   2646: 
                   2647: Комментарий:
                   2648: В 1939 году в доме Льюиса под Оксфордом жили несколько девочек,
                   2649: эвакуированных из Лондона во время войны. Льюис стал рассказывать им
                   2650: сказки: так жившие в его голове образы пришли в движение, а через
                   2651: несколько лет он понял, что рождающуюся историю необходимо записать.
                   2652: Схожим образом начинается и "Лев, колдунья и платяной шкаф": главных
                   2653: героев так же эвакуируют из Лондона и т.д. Занятно, что схожим же
                   2654: образом на свет появились и другие работы известных оксфордских
                   2655: профессоров - истории про хоббита и Алису.
                   2656: 
                   2657: Источник:
                   2658: https://arzamas.academy/mag/397-narnia
                   2659: 
                   2660: Автор:
                   2661: Алексей Кудяков
                   2662: 
                   2663: Вопрос 4:
                   2664: В 1817 году алжирский правитель Али бен Ахмед, опасаясь заговоров,
                   2665: покинул дворец и с группой приспешников заперся в отдаленной крепости.
                   2666: От чего умер Али бен Ахмед?
                   2667: 
                   2668: Ответ:
                   2669: От чумы.
                   2670: 
                   2671: Комментарий:
                   2672: В 1824 году Эдгар Аллан По написал один из самых известных своих
                   2673: рассказов - "Маска Красной Смерти". По сюжету рассказа принц с
                   2674: некоторыми придворными запирается в замке, чтобы избежать бушующей в
                   2675: стране болезни, однако в итоге та всё же проникает за стены. По мнению
                   2676: некоторых исследователей, Красная Смерть из рассказа - не что иное, как
                   2677: чума. В короткой и печальной истории взлета и падения Али бен Ахмеда
                   2678: несложно проследить параллели с рассказом. Учитывая близость этих
                   2679: событий на временной оси, можно предположить, что именно этой историей
                   2680: По в числе прочего вдохновлялся при написании рассказа.
                   2681: 
                   2682: Источник:
                   2683:    1. https://ru.wikipedia.org/wiki/Али_бен_Ахмед
                   2684:    2. https://ru.wikipedia.org/wiki/Маска_Красной_смерти
                   2685: 
                   2686: Автор:
                   2687: Алексей Кудяков
                   2688: 
                   2689: Вопрос 5:
                   2690: (pic: 20170795.jpg)
                   2691:    Перед вами кадр из мистического мультсериала "Грэвити Фоллз",
                   2692: действие которого происходит в Орегоне. Назовите вымышленный город,
                   2693: расположенный в соседнем штате.
                   2694: 
                   2695: Ответ:
                   2696: Твин Пикс.
                   2697: 
                   2698: Комментарий:
                   2699: Мультсериал рассказывает о небольшом городке Грэвити Фоллз, окруженном
                   2700: лесами. В Грэвити Фоллз происходят загадочные и необъяснимые события -
                   2701: точно так же, как и в другом вымышленном небольшом городке, окруженном
                   2702: лесами. Занятно, что оба города находятся в соседних штатах (Орегон и
                   2703: Вашингтон). На картинке можно увидеть леса и горы в отдалении: возможно,
                   2704: что среди них есть и те самые twin peaks.
                   2705: 
                   2706: Источник:
                   2707:    1. Наблюдения автора вопроса и его знакомой.
                   2708:    2. https://en.wikipedia.org/wiki/Gravity_Falls
                   2709:    3. https://en.wikipedia.org/wiki/Twin_Peaks
                   2710: 
                   2711: Автор:
                   2712: Алексей Кудяков
                   2713: 
                   2714: Вопрос 6:
                   2715: НХЛ девяностых была не такой вегетарианской лигой, как сейчас, а
                   2716: предпочитала мясо, желательно самой слабой прожарки. Так автор одной
                   2717: статьи объяснил прозвище, данное хоккеисту Владимиру Константинову.
                   2718: Напишите это прозвище.
                   2719: 
                   2720: Ответ:
                   2721: [Влад] Дракула.
                   2722: 
                   2723: Зачет:
                   2724: Колосажатель.
                   2725: 
                   2726: Комментарий:
                   2727: В то время в НХЛ было больше силовых приемов, а жесткие защитники были в
                   2728: большом почете. Игрок "Детройт Ред Уингз" Владимир Константинов был
                   2729: одним из сильнейших представителей своего амплуа. За его "Кровожадность"
                   2730: он и получил такое прозвище.
                   2731: 
                   2732: Источник:
                   2733: https://www.sports.ru/tribuna/blogs/centre/1302000.html
                   2734: 
                   2735: Автор:
                   2736: Григорий Смыслов
                   2737: 
                   2738: Вопрос 7:
                   2739: Телепередача "Орел и решка" часто сталкивается с трудностями во время
                   2740: записи сюжетов из-за запрета на съемку в некоторых общественных местах.
                   2741: При съемках эпизода в аэропорту, с которого начинается каждое
                   2742: путешествие, съемочная группа часто говорит полиции, что снимает...
                   2743: Ответьте двумя словами: что?
                   2744: 
                   2745: Ответ:
                   2746: Свадебное видео.
                   2747: 
                   2748: Зачет:
                   2749: Свадебное путешествие; медовый месяц.
                   2750: 
                   2751: Комментарий:
                   2752: В передаче "Орел и решка" двое ведущих - парень и девушка, по приезду в
                   2753: аэропорт они изображают счастливую пару.
                   2754: 
                   2755: Источник:
                   2756: Телепередача "Орел и решка. Перезагрузка: неизданное".
                   2757: 
                   2758: Автор:
                   2759: Дарья Бычкова
                   2760: 
                   2761: Вопрос 8:
                   2762: В журнале "GQ. Russia" была опубликована откровенная фотосессия
                   2763: украинской певицы Кристины Бардаш, также известной как "Луна". Заголовок
                   2764: перед фотосессией гласил: "АСТРОНОМИЧЕСКИЙ ОБЪЕКТ". Что мы заменили
                   2765: словами "АСТРОНОМИЧЕСКИЙ ОБЪЕКТ"?
                   2766: 
                   2767: Ответ:
                   2768: Небесное тело.
                   2769: 
                   2770: Источник:
                   2771: Журнал "GQ. Russia", май 2017 г.
                   2772: 
                   2773: Автор:
                   2774: Роман Трегуб
                   2775: 
                   2776: Вопрос 9:
                   2777: (pic: 20170796.jpg)
                   2778:    Напишите то, что мы скрыли на этой фотографии.
                   2779: 
                   2780: Ответ:
                   2781: Pho.
                   2782: 
                   2783: Комментарий:
                   2784: (pic: 20170797.jpg)
                   2785:    На фотографии - чехол для супницы, который сохраняет тепло и не
                   2786: позволяет обжечься о горячую посуду. "Фо" по-вьетнамски значит просто
                   2787: суп, а по-русски - новое кулинарное веяние. iPho созвучно с iPhone.
                   2788: 
                   2789: Источник:
                   2790: ЛОАВ.
                   2791: 
                   2792: Автор:
                   2793: Зарема Макаева
                   2794: 
                   2795: Тур:
                   2796: 7.2. "Во вторник сможем"
                   2797: 
                   2798: Дата:
                   2799: 19-May-2018
                   2800: 
                   2801: Редактор:
                   2802: Борис Белозёров (Москва)
                   2803: 
                   2804: Инфо:
                   2805: Редактор благодарит за тестирование и помощь в подготовке пакета: Марию
                   2806: Батутину, Татьяну Хадыеву, Сергея Гиршевича, Егора Мухина, Данилу
                   2807: Добровольского, а также Романа и Галину Царегородцевых.
                   2808: 
                   2809: Вопрос 1:
                   2810: [Нулевой вопрос]
                   2811:    <раздатка>
                   2812:    2. Dies Martis
                   2813:    &nbsp;
                   2814:    3. Одна фирма, выпускающая чашки необычного дизайна, утверждает, что
                   2815: их "чашки всегда протянут вам руку дружбы", но тут же исправляется,
                   2816: поскольку в данном случае речь может идти лишь об о.ь.ин..ь..
                   2817: рукопожатьи.
                   2818:    </раздатка>
                   2819:    Блиц.
                   2820:    1. Иногда при переводе на другой язык что-то теряется. Фамилия этого
                   2821: английского писателя в русском написании сократилась на шестьдесят
                   2822: процентов. Напишите его фамилию по-русски.
                   2823:    2. ИКС по-латыни - Dies Martis. Англичане же ставили в соответствие
                   2824: ИКСУ не Марса, а древнегерманского бога военной доблести Тиу. Назовите
                   2825: ИКС одним словом.
                   2826:    3. Какие буквы мы пропустили на раздаточном материале?
                   2827: 
                   2828: Ответ:
                   2829:    1. Во.
                   2830:    2. Вторник.
                   2831:    3. сможем.
                   2832: 
                   2833: Комментарий:
                   2834: Команда "Во вторник сможем" приветствует вас на своем туре Первой лиги
                   2835: Москвы!
                   2836: 
                   2837: Источник:
                   2838:    1. https://ru.wikipedia.org/wiki/Во,_Ивлин
                   2839:    2. http://www.bible.com.ua/answers/r/35/301941
                   2840:    3. https://lmbd.ru/jarjeurar/9-chashki-osminozhi-schypaltsa/
                   2841: 
                   2842: Автор:
                   2843: Андрей Дуков (Москва - Тамбов)
                   2844: 
                   2845: Вопрос 2:
                   2846: Сотрудники одного из сервисов Яндекса выяснили, что попасть НАВЕРХ можно
                   2847: за 15 часов 40 минут. Какие четыре слова в этом вопросе мы заменили
                   2848: одним?
                   2849: 
                   2850: Ответ:
                   2851: Из Грязи в Князи.
                   2852: 
                   2853: Зачет:
                   2854: С произвольной капитализацией.
                   2855: 
                   2856: Комментарий:
                   2857: Из города Грязи Липецкой области до деревни Князи.
                   2858: 
                   2859: Источник:
                   2860: https://vk.com/nmaps_official?w=wall-135388977_1332
                   2861: 
                   2862: Автор:
                   2863: Константин Шведов (Москва)
                   2864: 
                   2865: Вопрос 3:
                   2866: Английское название одной инди-игры, в которой пользователю предлагается
                   2867: построить гигантский шоппинг-центр, одной буквой отличается от названия
                   2868: известной песни, вышедшей в 1979 году. Напишите название этой игры.
                   2869: 
                   2870: Ответ:
                   2871: "Another Brick In The Mall".
                   2872: 
                   2873: Комментарий:
                   2874: Игра как раз и представляет собой симулятор постройки торгового молла
                   2875: "по кирпичикам".
                   2876: 
                   2877: Источник:
                   2878:    1. http://store.steampowered.com/app/521150/Another_Brick_in_the_Mall/
                   2879:    2. https://ru.wikipedia.org/wiki/Another_Brick_in_the_Wall
                   2880: 
                   2881: Автор:
                   2882: Павел Семенюк (Москва - Гринфилд)
                   2883: 
                   2884: Вопрос 4:
                   2885: East Side Gallery в Нью-Йорке является самой длинной в мире
                   2886: художественной галереей под открытым небом с постоянной экспозицией.
                   2887: Какое слово мы заменили в тексте этого вопроса?
                   2888: 
                   2889: Ответ:
                   2890: Берлине.
                   2891: 
                   2892: Зачет:
                   2893: Берлин.
                   2894: 
                   2895: Комментарий:
                   2896: Эта галерея расположена не в нью-йоркском районе Ист-Сайд, а на
                   2897: восточной стороне Берлинской стены. Длина галереи составляет 1316
                   2898: метров, а одной из самых известных картин является работа Дмитрия
                   2899: Врубеля "Господи! Помоги мне выжить среди этой смертной любви".
                   2900: 
                   2901: Источник:
                   2902: https://ru.wikipedia.org/wiki/East_Side_Gallery
                   2903: 
                   2904: Автор:
                   2905: Павел Семенюк (Москва - Гринфилд)
                   2906: 
                   2907: Вопрос 5:
                   2908: Внимание, в вопросе слово "СУББОТА" заменяет другое слово.
                   2909:    Слоган манги "Йотсуба то" об очень жизнерадостной девочке гласит:
                   2910: "СУББОТА - самый счастливый день". Назовите то, что, согласно русскому
                   2911: переводу названия оскароносного фильма 1979 года, приходится на СУББОТУ.
                   2912: 
                   2913: Ответ:
                   2914: Апокалипсис.
                   2915: 
                   2916: Комментарий:
                   2917: Жизнерадостная девочка утверждает, что сегодня - самый счастливый день.
                   2918: На вручении премии киноакадемии 1980 года фильм "Апокалипсис сегодня",
                   2919: снятый в 1979 году, получил два "Оскара" - за лучшие звук и операторскую
                   2920: работу.
                   2921: 
                   2922: Источник:
                   2923:    1. https://en.wikipedia.org/wiki/Yotsuba%26!
                   2924:    2. https://ru.wikipedia.org/wiki/Апокалипсис_сегодня
                   2925: 
                   2926: Автор:
                   2927: Никита Корчагин (Москва - Ростов-на-Дону)
                   2928: 
                   2929: Вопрос 6:
                   2930: Статья на портале sports.ru [спортс точка ру] о футбольной
                   2931: школе-академии клуба "Краснодар" называется "Кубанский казачий Хор". В
                   2932: одном из слов предыдущего предложения мы пропустили пять букв.
                   2933: Восстановите это слово в исходном виде.
                   2934: 
                   2935: Ответ:
                   2936: Хогвартс.
                   2937: 
                   2938: Комментарий:
                   2939: Недаром статья посвящена "школе-академии".
                   2940: 
                   2941: Источник:
                   2942: https://www.sports.ru/tribuna/blogs/sportbizinfo/1169156.html
                   2943: 
                   2944: Автор:
                   2945: Павел Семенюк (Москва - Гринфилд)
                   2946: 
                   2947: Вопрос 7:
                   2948:    <раздатка>
                   2949:    "Клин клином?"
                   2950:    </раздатка>
                   2951:    Перед вами название статьи в "Российской газете", которая посвящена
                   2952: методике лечения алкоголизма наркотическими веществами, в котором мы
                   2953: дважды заменили две буквы одной. Согласно этой статье, у алкоголиков
                   2954: выход все-таки есть. Восстановите название статьи в исходном виде.
                   2955: 
                   2956: Ответ:
                   2957: Сплин сплином?
                   2958: 
                   2959: Комментарий:
                   2960: "Splean" в переводе с английского - хандра, ассоциирующаяся, видимо, у
                   2961: авторов статьи с алкоголем и наркотиками. Слова "выход все-таки есть"
                   2962: намекают на название одной из самых известных песен группы "Сплин" -
                   2963: "Выхода нет".
                   2964: 
                   2965: Источник:
                   2966: https://rg.ru/2017/07/13/v-velikobritanii-alkogolizm-predlozhili-lechit-narkotikami.html
                   2967: 
                   2968: Автор:
                   2969: Сергей Дуликов (Москва - Брянск)
                   2970: 
                   2971: Вопрос 8:
                   2972: В постапокалиптической короткометражке ОН неожиданно возвращается
                   2973: обратно. Благодаря этому главная героиня узнает о красоте былой природы
                   2974: из черно-белых фильмов и старых романсов. Назовите ЕГО.
                   2975: 
                   2976: Ответ:
                   2977: "Вояджер-1".
                   2978: 
                   2979: Зачет:
                   2980: По слову "Вояджер".
                   2981: 
                   2982: Комментарий:
                   2983: На золотом диске, находящемся на борту космического аппарата
                   2984: "Вояджер-1", записаны черно-белые фильмы, романсы и фотографии природы.
                   2985: 
                   2986: Источник:
                   2987: https://www.mirf.ru/media/korotkometrazhka-voyager
                   2988: 
                   2989: Автор:
                   2990: Андрей Дуков (Москва - Тамбов), в редакции Марии Батутиной (Москва -
                   2991: Архангельск)
                   2992: 
                   2993: Вопрос 9:
                   2994: Валентин Пикуль отмечает, что поселенцы Сахалина и его исконные
                   2995: обитатели были друг для друга в диковинку, поэтому овцы при виде волков
                   2996: в страхе разбегались во все стороны. Какие два слова мы заменили в
                   2997: предыдущем предложении?
                   2998: 
                   2999: Ответ:
                   3000: Волки, овец.
                   3001: 
                   3002: Комментарий:
                   3003: Не овцы при виде волков, а волки при виде овец.
                   3004: 
                   3005: Источник:
                   3006: В.С. Пикуль. Каторга. http://flibusta.is/b/361630/read
                   3007: 
                   3008: Автор:
                   3009: Андрей Дуков (Москва - Тамбов)
                   3010: 
                   3011: Вопрос 10:
                   3012: В пародии "Большой разницы" Геннадий Малахов, будучи гостем программы
                   3013: "Модный приговор", предлагает ведущему надеть НЕПРОСТОЕ УКРАШЕНЬЕ, ведь
                   3014: это помогает от морщин. Автор вопроса назвал НЕПРОСТОЕ УКРАШЕНЬЕ
                   3015: бюджетным вариантом для больных глаукомой. Какие два слова, начинающиеся
                   3016: на одну и ту же букву, мы заменили словами "НЕПРОСТОЕ УКРАШЕНЬЕ"?
                   3017: 
                   3018: Ответ:
                   3019: Огуречные очки.
                   3020: 
                   3021: Зачет:
                   3022: Огурцовые очки.
                   3023: 
                   3024: Комментарий:
                   3025: Малахов известен своей любовью к традиционной медицине. В частности, он
                   3026: как герой пародии советует носить очки из огурцов, помогающие от морщин.
                   3027: Глаукома могла намекнуть вам на глаза, а непосредственно от глаукомы
                   3028: помогают зеленые очки. Замена обусловлена эквиритмичностью, а также
                   3029: связана с популярностью в последний год в ЧГК-среде соответствующего
                   3030: мема.
                   3031: 
                   3032: Источник:
                   3033: "Большая разница: Круговорот Геннадия Малахова".
                   3034: https://www.youtube.com/watch?v=fg58ZhpaOf4
                   3035: 
                   3036: Автор:
                   3037: Павел Семенюк (непростое украшенье)
                   3038: 
                   3039: Тур:
                   3040: 8.1. "Искусство ухода"
                   3041: 
                   3042: Дата:
                   3043: 19-May-2018
                   3044: 
                   3045: Редактор:
                   3046: Роман Царегородцев
                   3047: 
                   3048: Инфо:
                   3049: Редактор благодарит за помощь в подготовке пакета Наталью Сычёву,
                   3050: Михаила Лаврина, Галину Царегородцеву, Дениса Иванова, Якова Летучего,
                   3051: Викторию Полякову, Татьяну Хадыеву, Сергея Гиршевича, Данилу
                   3052: Добровольского.
                   3053: 
                   3054: Вопрос 1:
                   3055: Мы бы хотели задать этот вопрос завтра.
                   3056:    2 января 1955 года в Австралии была выжжена земля на площади в 600
                   3057: квадратных миль. Этот день был назван ТАКИМ ИМ. Как правило, ТАКОЕ ОНО в
                   3058: большинстве стран не связывают с событиями на финансовых рынках. Какие
                   3059: два слова мы заменили словами "ТАКОЕ ОНО"?
                   3060: 
                   3061: Ответ:
                   3062: Черное воскресенье.
                   3063: 
                   3064: Комментарий:
                   3065: "Черными" днями недели называют значительное падение биржевых котировок.
                   3066: В воскресенье большинство бирж не работает, поэтому обвал рынков в
                   3067: воскресенье случается крайне редко. Австралийское "черное воскресенье"
                   3068: получило свое название из-за черного цвета выжженной пожаром земли. А с
                   3069: учетом качества наших вопросов любой день будет для вас черным.
                   3070: 
                   3071: Источник:
                   3072:    1. http://www.sahistorians.org.au/175/chronology/january/2-january-1955-black-sunday.shtml
                   3073:    2. https://en.wikipedia.org/wiki/Black_Sunday
                   3074: 
                   3075: Автор:
                   3076: Алексей Овчинников
                   3077: 
                   3078: Вопрос 2:
                   3079: В вопросе есть сложные замены.
                   3080:    Украшения, которые, согласно рекламе, показывали ИКС носителя, на
                   3081: самом деле реагировали на температуру тела. Согласно описанию украшений,
                   3082: ТАКОЙ ИКС соответствовал спокойствию и расслабленности. Какие слова мы
                   3083: заменили на "ИКС ТАКОЙ"?
                   3084: 
                   3085: Ответ:
                   3086: Цвет настроения синий.
                   3087: 
                   3088: Комментарий:
                   3089: Популярные в конце прошлого века "кольца настроения" якобы отражали
                   3090: внутреннее состояние владельца. Синий цвет кольца по задумке создателей
                   3091: означал, что владелец спокоен. Надеемся, что игроки уже успели услышать
                   3092: новый хит Филиппа Киркорова - "Цвет настроения синий".
                   3093: 
                   3094: Источник:
                   3095: https://www.thoughtco.com/mood-ring-colors-and-meanings-608026
                   3096: 
                   3097: Автор:
                   3098: Юлия Колодяжная, Юлия Лукьянова
                   3099: 
                   3100: Вопрос 3:
                   3101: (pic: 20170798.jpg)
                   3102:    Восстановите любую из надписей легенды этого графика из статьи
                   3103: Википедии.
                   3104: 
                   3105: Ответ:
                   3106: Internet Explorer.
                   3107: 
                   3108: Зачет:
                   3109: Mozilla Firefox; Google Chrome; Safari; Opera.
                   3110: 
                   3111: Комментарий:
                   3112: Этот график отражает процент использования различных веб-браузеров.
                   3113: Цвета браузеров и линий на графике схожи.
                   3114: 
                   3115: Источник:
                   3116: https://en.wikipedia.org/wiki/Usage_share_of_web_browsers
                   3117: 
                   3118: Автор:
                   3119: Иван Петренко
                   3120: 
                   3121: Вопрос 4:
                   3122: Страница значений некоторого слова в англоязычной Википедии содержит,
                   3123: например, ссылки на статьи про эссе РолАна БАрта "Смерть автора", фильм
                   3124: "День животных" или певицу Доротею Кер, а основным значением этого слова
                   3125: считается нечто, появившееся на свет в 2002 году. Напишите это слово.
                   3126: 
                   3127: Ответ:
                   3128: Dota.
                   3129: 
                   3130: Комментарий:
                   3131: Оригинальные названия упомянутых фильма и эссе - "Day of the Animals" и
                   3132: "Death of the Author" - имеют аббревиатуры, совпадающие с названием
                   3133: популярной игры. Доротея Кер выступает под сокращенной версией своего
                   3134: имени.
                   3135: 
                   3136: Источник:
                   3137:    1. https://en.wikipedia.org/wiki/Dota_(disambiguation)
                   3138:    2. https://en.wikipedia.org/wiki/Defense_of_the_Ancients
                   3139: 
                   3140: Автор:
                   3141: Иван Петренко
                   3142: 
                   3143: Вопрос 5:
                   3144: Болезнь ИХ протекает с поражением центральной нервной системы, легких, а
                   3145: иногда и с полиОрганной недостаточностью, так что вы вряд ли сможете
                   3146: играть или сражаться. Назовите ИХ.
                   3147: 
                   3148: Ответ:
                   3149: Легионеры.
                   3150: 
                   3151: Комментарий:
                   3152: Клинический вариант легионеллёза, также называемый "болезнью
                   3153: легионеров", поражает сразу несколько систем органов, что определенно
                   3154: мешает выполнять свои обязанности как легионеру-воину, так и
                   3155: легионеру-чгкшнику.
                   3156: 
                   3157: Источник:
                   3158: https://ru.wikipedia.org/wiki/Легионеллёз
                   3159: 
                   3160: Автор:
                   3161: Артем Земцовский
                   3162: 
                   3163: Вопрос 6:
                   3164: Коготь одной из моделей, расположенных в старом музее под открытым небом
                   3165: в южной части Лондона, приделан на нос, сама же модель стоит на четырех
                   3166: ногах вместо двух. По словам обозревателя газеты "Independent"
                   3167: [индепЕндент], данный музей - первый в мире [ПРОПУСК]. Заполните пропуск
                   3168: названием произведения 1993 года.
                   3169: 
                   3170: Ответ:
                   3171: Парк Юрского периода.
                   3172: 
                   3173: Комментарий:
                   3174: В парке Хрустального дворца была представлена первая в мире выставка
                   3175: моделей динозавров под открытым небом. На заре изучения этих древних
                   3176: ящеров палеонтологи часто совершали ошибки.
                   3177: 
                   3178: Источник:
                   3179:    1. Б. Брайсон. Краткая история почти всего на свете.
                   3180: http://flibusta.is/b/512731/read
                   3181:    2. https://www.independent.co.uk/travel/travel-swot-spot-monsters-escape-from-movie-you-dont-have-to-see-jurassic-park-to-learn-all-about-1486886.html
                   3182:    3. https://ru.wikipedia.org/wiki/Парк_юрского_периода_(фильм)
                   3183: 
                   3184: Автор:
                   3185: Артем Земцовский
                   3186: 
                   3187: Вопрос 7:
                   3188: На одной лекции по супрамолекулярной химии рассказывалось о том, что
                   3189: огромную роль здесь играет большое количество слабых связей. Иллюстрируя
                   3190: это, преподаватель упомянул сцену из одного произведения. Назовите
                   3191: заглавного героя этого произведения.
                   3192: 
                   3193: Ответ:
                   3194: Гулливер.
                   3195: 
                   3196: Комментарий:
                   3197: При подобном связывании молекул проявляется так называемый "эффект
                   3198: Гулливера". Когда лилипуты связали Гулливера множеством тонких канатов,
                   3199: он не мог пошевелиться.
                   3200: 
                   3201: Источник:
                   3202: http://www.nanonewsnet.ru/articles/2010/supramolekulyarnye-sistemy-most-mezhdu-nezhivoi-zhivoi-materiei
                   3203: 
                   3204: Автор:
                   3205: Юлия Колодяжная
                   3206: 
                   3207: Вопрос 8:
                   3208: (pic: 20170799.jpg)
                   3209:    Перед вами две картины современного художника Алексея Уварова.
                   3210: Название первой картины отличается от надписи, которую мы закрыли на
                   3211: второй картине, одним слогом. Восстановите закрашенную надпись.
                   3212: 
                   3213: Ответ:
                   3214: Гигиена огненная.
                   3215: 
                   3216: Комментарий:
                   3217: Первая картина называется "Гиена огненная", в ней автор обыгрывает
                   3218: библейское понятие "геенна огненная". Вторая картина изображает
                   3219: Мойдодыра - заглавного героя сказки, пропагандирующей чистоту.
                   3220: 
                   3221: Источник:
                   3222:    1. https://arayvo.livejournal.com/5641652.html
                   3223:    2. https://arayvo.livejournal.com/5628948.html
                   3224: 
                   3225: Автор:
                   3226: Николай Дорофеев
                   3227: 
                   3228: Вопрос 9:
                   3229: Дорожную табличку, встретившуюся автору вопроса в горной части Крыма,
                   3230: можно благодаря чьей-то шутке понять как предупреждение о появлении на
                   3231: дороге маленькой серой птички. Напишите опасность, о которой
                   3232: предупреждал знак изначально.
                   3233: 
                   3234: Ответ:
                   3235: Оползень.
                   3236: 
                   3237: Комментарий:
                   3238: Ну а птичка, соответственно, - поползень.
                   3239: 
                   3240: Источник:
                   3241: Личный опыт автора вопроса.
                   3242: 
                   3243: Автор:
                   3244: Мария Петрова
                   3245: 
                   3246: Тур:
                   3247: 8.2. "Тошнота от Смирновской"
                   3248: 
                   3249: Дата:
                   3250: 19-May-2018
                   3251: 
                   3252: Редактор:
                   3253: Николай Некрылов (Москва)
                   3254: 
                   3255: Инфо:
                   3256: Автор благодарит за помощь в подготовке пакета Антона Волосатова, Карину
                   3257: Файзуллину, Павла Казначеева, Дарью Семёнову, Маргариту Махрову, Вадима
                   3258: Штанникова, Леонида Мужикбаева, Юлию Кожевникову, Артура Кадурина, Илью
                   3259: Бодрова, Арсения Шумилова, Филиппа Терёхина, Анатолия Барковского, Юрия
                   3260: Фокина и Дмитрия Осипкова, который единственный тестировал вопросы
                   3261: трезвым. Руслана Хаиткулова, который большую часть тестов спал, автор
                   3262: тоже благодарит, но так, меньше, чем остальных. Этот пакет не
                   3263: рекомендован к употреблению людям, не достигшим 18 лет, а также
                   3264: обладающим тонкой душевной организацией.
                   3265: 
                   3266: Вопрос 1:
                   3267: [Нулевой вопрос]
                   3268:    В фильме 2017 года герой переезжает из США в Италию и начинает
                   3269: заниматься ИМ, так как привык использовать руки. Назовите ЕГО двумя
                   3270: словами, начинающимися на одну и ту же букву.
                   3271: 
                   3272: Ответ:
                   3273: Флорентийский футбол.
                   3274: 
                   3275: Комментарий:
                   3276: Во Флоренции есть старинное соревнование - "кальчо", которое также
                   3277: называют флорентийским футболом. В него, в отличие от обычного футбола,
                   3278: играют руками.
                   3279:    Филипп Терёхин настаивал на том, чтобы в этом отвратительном пакете
                   3280: обязательно был отвратительный вопрос про футбол. Это был отвратительный
                   3281: вопрос про футбол.
                   3282: 
                   3283: Источник:
                   3284: Х/ф "Потерявшийся во Флоренции" (2017), реж. Эван Оппенгеймер.
                   3285: 
                   3286: Автор:
                   3287: Николай Некрылов (Москва)
                   3288: 
                   3289: Вопрос 2:
                   3290: [Нулевой вопрос]
                   3291:    По мнению автора вопроса, рано или поздно должна появиться пародия на
                   3292: фильм "Сияние", где в главной роли будет этот грешник. Назовите его имя
                   3293: и фамилию.
                   3294: 
                   3295: Ответ:
                   3296: Джонни Синс.
                   3297: 
                   3298: Комментарий:
                   3299: Джонни Синс - тот самый лысый из Браззерс, который сыграл в кино уже
                   3300: много ролей. Каким прибором он будет вырубать дверь в знаменитой сцене,
                   3301: пока остается загадкой, однако фразу "Here is Johnny" он в этом случае
                   3302: произнесет наверняка. Этой фразой наша команда приветствует участников
                   3303: Первой лиги.
                   3304: 
                   3305: Источник:
                   3306: Логические соображения автора вопроса.
                   3307: 
                   3308: Автор:
                   3309: Николай Некрылов (Москва)
                   3310: 
                   3311: Вопрос 3:
                   3312: По признанию Стэнли Кубрика, классический комедийный фильм "Доктор
                   3313: Стрейнджлав, или Как я перестал бояться и полюбил бомбу" является
                   3314: метафорой полового акта. В оригинальной концовке фильма, от которой
                   3315: Кубрик впоследствии отказался, в штабе устраивают битву ИМИ. Назовите ИХ
                   3316: по-английски или по-русски.
                   3317: 
                   3318: Ответ:
                   3319: Cream pies.
                   3320: 
                   3321: Зачет:
                   3322: Кремовые пироги; кремовые торты.
                   3323: 
                   3324: Комментарий:
                   3325: Если весь фильм воспринимать как метафору полового акта, то эта концовка
                   3326: выглядит весьма логичной, к тому же кидание друг в друга кремовых
                   3327: пирогов является классическим комедийным приемом. Если вы всё еще не
                   3328: понимаете этот вопрос, то вы в своей жизни просмотрели недостаточно
                   3329: порнографии.
                   3330: 
                   3331: Источник:
                   3332: https://en.wikipedia.org/wiki/Dr._Strangelove#Sexual_themes
                   3333: 
                   3334: Автор:
                   3335: Николай Некрылов (Москва)
                   3336: 
                   3337: Вопрос 4:
                   3338: Как Алистер Кроули назвал свою первую дочь?
                   3339: 
                   3340: Ответ:
                   3341: Лилит.
                   3342: 
                   3343: Зачет:
                   3344: Нуит; Ма; Ахатор; Геката; Сафо; Джезабел.
                   3345: 
                   3346: Комментарий:
                   3347: Алистер Кроули дал своей первой дочери имя Нуит Ма Ахатор Геката Сафо
                   3348: Джезабел Лилит, однако называли ее просто Лилит, как первую жену Адама,
                   3349: - подходящее имя для дочери одного из самых известных оккультистов.
                   3350: Впрочем, вторую его дочь звали не Ева, а Лола Заза.
                   3351: 
                   3352: Источник:
                   3353: https://en.wikipedia.org/wiki/Aleister_Crowley
                   3354: 
                   3355: Автор:
                   3356: Николай Некрылов (Москва)
                   3357: 
                   3358: Вопрос 5:
                   3359: Эмигрировав из Британии в США в 1914 году, Алистер Кроули вел не самую
                   3360: образцовую жизнь, пользуясь услугами проституток, находя любовников в
                   3361: турецких банях и открыто поддерживая Германию в Первой мировой войне.
                   3362: Впрочем, последнее оправдывает то, что Кроули был ИМ. Назовите ЕГО двумя
                   3363: словами.
                   3364: 
                   3365: Ответ:
                   3366: Двойной агент.
                   3367: 
                   3368: Комментарий:
                   3369: Кроули работал на британскую разведку и выполнял задачу внедрения в ряды
                   3370: прогерманских активистов. В сексе он тоже играл за обе команды.
                   3371: 
                   3372: Источник:
                   3373: https://en.wikipedia.org/wiki/Aleister_Crowley
                   3374: 
                   3375: Автор:
                   3376: Николай Некрылов (Москва)
                   3377: 
                   3378: Вопрос 6:
                   3379: В фильме "Тупой и еще тупее - 2" дочь обращается к отцу-одиночке с
                   3380: деликатной проблемой. Увидев аналогию, он предлагает дочери использовать
                   3381: ЕЕ. Назовите ЕЕ как можно точнее.
                   3382: 
                   3383: Ответ:
                   3384: Пробка от бутылки с красным вином.
                   3385: 
                   3386: Зачет:
                   3387: По смыслу.
                   3388: 
                   3389: Комментарий:
                   3390: У девочки началась менструация, а аналогия очевидна.
                   3391: 
                   3392: Источник:
                   3393: Х/ф "Тупой и еще тупее - 2" (2014), реж. Питер Фаррелли, Бобби Фаррелли.
                   3394: 
                   3395: Автор:
                   3396: Николай Некрылов (Москва)
                   3397: 
                   3398: Вопрос 7:
                   3399: Джон Оливер, рассуждая об отвратительной ситуации с коррупцией в FIFA,
                   3400: упоминает один кубок мира и ИХ. Назовите ИХ двумя словами, начинающимися
                   3401: на одну и ту же букву.
                   3402: 
                   3403: Ответ:
                   3404: Две девушки.
                   3405: 
                   3406: Комментарий:
                   3407: Джон Оливер говорил о кубке мира, проходившем в Бразилии, но вряд ли
                   3408: ситуация будет чем-либо отличаться на кубке мира в России. Если вы всё
                   3409: еще не понимаете этот вопрос, то вы в своей жизни просмотрели
                   3410: недостаточно порнографии.
                   3411: 
                   3412: Источник:
                   3413: Last Week Tonight, "FIFA and the World Cup".
                   3414: https://www.youtube.com/watch?v=DlJEt2KU33I
                   3415: 
                   3416: Автор:
                   3417: Николай Некрылов (Москва)
                   3418: 
                   3419: Вопрос 8:
                   3420: В эпизоде одного фильма солдат рассказывает, что была, наверное,
                   3421: единственной девственницей в НЕМ. Назовите ЕГО заимствованным словом.
                   3422: 
                   3423: Ответ:
                   3424: Кибуц.
                   3425: 
                   3426: Комментарий:
                   3427: Фильм, как можно догадаться, израильский. А о русскоязычных шутках про
                   3428: то, что делают в кибуцах, вы могли узнать, к примеру, из песен Сергея и
                   3429: Татьяны Никитиных.
                   3430: 
                   3431: Источник:
                   3432: Х/ф "Мотивации ноль" (2014), реж. Талья Лави.
                   3433: 
                   3434: Автор:
                   3435: Николай Некрылов (Москва)
                   3436: 
                   3437: Вопрос 9:
                   3438: Согласно англоязычной шутке, врачи говорят о стабильном состоянии
                   3439: человека, который был доставлен в больницу с несколькими игрушечными ИМИ
                   3440: в заднем проходе. Назовите ИХ одним словом.
                   3441: 
                   3442: Ответ:
                   3443: Лошади.
                   3444: 
                   3445: Зачет:
                   3446: Лошадки, пони, поняши, поняшки и т.п.
                   3447: 
                   3448: Комментарий:
                   3449: Его состояние - "stable", что можно перевести и как "стабильное", и как
                   3450: "конюшня".
                   3451: 
                   3452: Источник:
                   3453: Шутейка из Интернета.
                   3454: 
                   3455: Автор:
                   3456: Николай Некрылов (Москва)
                   3457: 
                   3458: Вопрос 10:
                   3459:    <раздатка>
                   3460:    ... fight
                   3461:    </раздатка>
                   3462:    В одной из серий мультсериала "Симпсоны" Мардж в борьбе за
                   3463: нравственность выходит на ринг боев без правил. Ринг-анонсер называет ее
                   3464: словосочетанием, в котором мы пропустили несколько слов. Напишите эти
                   3465: слова.
                   3466: 
                   3467: Ответ:
                   3468: Mom I'd like to.
                   3469: 
                   3470: Зачет:
                   3471: Mother I'd like to; другие варианты с небольшими искажениями.
                   3472: 
                   3473: Комментарий:
                   3474: Мардж является привлекательной молодой мамой Барта и Лизы, а ее образ не
                   3475: раз сексуализировался и в самих "Симпсонах". Если вы всё еще не
                   3476: понимаете этот вопрос, то вы в своей жизни просмотрели недостаточно
                   3477: порнографии.
                   3478: 
                   3479: Источник:
                   3480: Мультсериал "Симпсоны", s21e03.
                   3481: 
                   3482: Автор:
                   3483: Николай Некрылов (Москва)
                   3484: 
                   3485: Вопрос 11:
                   3486: Диалог пары героев одного комедийного мультсериала, проводящих
                   3487: романтический вечер в ресторане, периодически прерывается возгласами
                   3488: официанта, который вежливо приветствует входящих Керри Андервуд, Кери
                   3489: Маллиган и Марайю Керри. Ответьте одним словом: о чем не решается
                   3490: героиня этого мультсериала сообщить возлюбленному?
                   3491: 
                   3492: Ответ:
                   3493: Выкидыш.
                   3494: 
                   3495: Комментарий:
                   3496: К каждой официант вежливо обращается "мисс", а к Марайе Керри еще и по
                   3497: фамилии, что созвучно английскому слову "miscarriage", обозначающему
                   3498: выкидыш. Наш тур начался с кремового пирога, то есть, с определенной
                   3499: долей вероятности, с зачатия, и логично завершился выкидышем.
                   3500: 
                   3501: Источник:
                   3502: Мультсериал "Конь Боджек", s04e09.
                   3503: 
                   3504: Автор:
                   3505: Николай Некрылов (Москва)
                   3506: 
                   3507: Тур:
                   3508: 9.1. "Зеленый индийский"
                   3509: 
                   3510: Дата:
                   3511: 19-May-2018
                   3512: 
1.2       rubashki 3513: Редактор:
                   3514: Серафим Шибанов (Москва)
                   3515: 
1.1       rubashki 3516: Вопрос 1:
                   3517: (pic: 20170800.jpg)
                   3518:    Согласно шутке с сайта pikabu.ru [пикабУ точка ру] на выданном вам
                   3519: раздаточном материале изображен тренировочный лагерь ИХ. Назовите ИХ
                   3520: двумя словами.
                   3521: 
                   3522: Ответ:
                   3523: Свидетели Иеговы.
                   3524: 
                   3525: Источник:
                   3526: https://pikabu.ru/story/quottrenirovochnyiy_lager_svideteley_iegovyikhquot_3406066
                   3527: 
                   3528: Автор:
                   3529: Алексей Григорьев
                   3530: 
                   3531: Вопрос 2:
                   3532: "Олимпийские игры" - это название одной статьи об использовании
                   3533: спортсменами допинга. В этом названии мы чуть изменили одно слово.
                   3534: Напишите его в исходном виде.
                   3535: 
                   3536: Ответ:
                   3537: Иглы.
                   3538: 
                   3539: Комментарий:
                   3540: "Олимпийские иглы".
                   3541: 
                   3542: Источник:
                   3543: http://expert.ru/russian_reporter/2015/08/olimpijskie-iglyi/
                   3544: 
                   3545: Автор:
                   3546: Алексей Григорьев
                   3547: 
                   3548: Вопрос 3:
                   3549: Закончите стихотворение-порошок тремя словами:
                   3550:    Я, опровергнув поговорку,
                   3551:    Познал: нет истины в вине!
                   3552:    Есть лишь один осадок ...
                   3553:    ...
                   3554: 
                   3555: Ответ:
                   3556: Горький на дне.
                   3557: 
                   3558: Зачет:
                   3559: С любой пунктуацией.
                   3560: 
                   3561: Источник:
                   3562: https://vk.com/rhymesee?w=wall-161180646_34966
                   3563: 
                   3564: Автор:
                   3565: Алексей Григорьев
                   3566: 
                   3567: Вопрос 4:
                   3568: В Средние века остров Мэн страдал от непрекращающихся пиратских набегов.
                   3569: Не имея сил на вооруженное сопротивление, островитяне шли на
                   3570: разнообразные хитрости, чтобы обезопасить хотя бы часть своего имущества
                   3571: в случае внезапного нападения. С тех времен пошла традиция, по которой
                   3572: ПЕРВОЕ идет последним. Назовите ПЕРВОЕ.
                   3573: 
                   3574: Ответ:
                   3575: Суп.
                   3576: 
                   3577: Комментарий:
                   3578: В большинстве привычных нам культур суп едят первым, но на острове Мэн
                   3579: первыми едят более ценные рыбу или мясо.
                   3580: 
                   3581: Источник:
                   3582: Память острова Мэн. Книга сказаний. - М.-СПб.: Летний сад, 2002.
                   3583: 
                   3584: Автор:
                   3585: Илья Спектор
                   3586: 
                   3587: Вопрос 5:
                   3588: В конце 1930-х годов в Калькутте было основано "Научное общество по
                   3589: изучению звезд", членами которого стали многие представители индийской
                   3590: аристократии. Одним из основных пунктов расходов общества была покупка
                   3591: билетов на трансконтинентальные перелеты. Какое слово мы пропустили в
                   3592: первом предложении?
                   3593: 
                   3594: Ответ:
                   3595: Голливуда.
                   3596: 
                   3597: Комментарий:
                   3598: Вы могли бы подумать, что так зарождался Болливуд, но ездили они просто
                   3599: для развлечения.
                   3600: 
                   3601: Источник:
                   3602: http://www.mountain.ru/article/article_display1.php?article_id=7416
                   3603: 
                   3604: Автор:
                   3605: Илья Спектор
                   3606: 
                   3607: Вопрос 6:
                   3608: В казачьих станицах на Урале, где большую роль играло ОНО, в
                   3609: определенные периоды времени был запрещен колокольный звон в храмах. С
                   3610: НИМ связан один из главных символов власти... Чьей именно власти?
                   3611: 
                   3612: Ответ:
                   3613: Папы Римского.
                   3614: 
                   3615: Комментарий:
                   3616: ОНО - рыболовство. Символ - "кольцо рыбака".
                   3617: 
                   3618: Источник:
                   3619:    1. https://ru.wikipedia.org/wiki/Уральские_казаки
                   3620:    2. https://ru.wikipedia.org/wiki/Кольцо_рыбака
                   3621: 
                   3622: Автор:
                   3623: Илья Спектор
                   3624: 
                   3625: Вопрос 7:
                   3626: По мнению эксперта Ивана Кузнецова, с появлением криптовалюты ТАКУЮ
                   3627: АЛЬФУ сменила анонимная. Назовите ТАКУЮ АЛЬФУ тремя словами.
                   3628: 
                   3629: Ответ:
                   3630: Невидимая рука рынка.
                   3631: 
                   3632: Источник:
                   3633: ???
                   3634: 
                   3635: Автор:
                   3636: Александр Сабреков
                   3637: 
                   3638: Вопрос 8:
                   3639: Генри Тенди неоднократно выражал сожаление, что не воспользовался
                   3640: возможностью СДЕЛАТЬ ЭТО, которая якобы представилась ему в конце
                   3641: сентября 1918 года. Более поздняя попытка СДЕЛАТЬ ЭТО провалилась из-за
                   3642: низкой температуры в багажном отделении. Какие два слова мы заменили
                   3643: словами "СДЕЛАТЬ ЭТО"?
                   3644: 
                   3645: Ответ:
                   3646: Убить Гитлера.
                   3647: 
                   3648: Комментарий:
                   3649: 3 марта 1943 года во время посещения Адольфом Гитлером группы армий
                   3650: "Центр" в самолет фюрера под видом посылки заложили бомбу (операция
                   3651: "Вспышка"). Она должна была взорваться во время возвращения Гитлера на
                   3652: самолете из Смоленска в Берлин. Но взрыватель не сработал. Есть версия,
                   3653: что он не сработал из-за слишком низкой температуры в багажном отделении
                   3654: самолета.
                   3655: 
                   3656: Источник:
                   3657:    1. https://mi3ch.livejournal.com/2271356.html
                   3658:    2. http://www.istpravda.ru/pictures/9953/
                   3659: 
                   3660: Автор:
                   3661: Илья Спектор
                   3662: 
                   3663: Вопрос 9:
                   3664: По словам Джастина Ройланда, в "Рике и Морти" никогда не будет ЭТОГО. В
                   3665: романе Александра Вельтмана "Александр Филиппович Македонский. Предки
                   3666: Калимероса" (1836) для ЭТОГО используется волшебный гиппогриф. Назовите
                   3667: ЭТО.
                   3668: 
                   3669: Ответ:
                   3670: Путешествие во времени.
                   3671: 
                   3672: Комментарий:
                   3673: Несмотря на то что мультсериал "Рик и Морти" является своеобразной
                   3674: пародией на "Назад в будущее", создатели планируют избегать этого приема
                   3675: во избежание возникновения нестыковок в сюжете.
                   3676: 
                   3677: Источник:
                   3678:    1. https://www.2x2tv.ru/blog/rick-and-morty-facts/
                   3679:    2. https://ru.wikipedia.org/wiki/Путешествие_во_времени
                   3680: 
                   3681: Автор:
                   3682: Алексей Григорьев
                   3683: 
                   3684: Тур:
                   3685: 9.2. Оргкомитет
                   3686: 
                   3687: Дата:
                   3688: 19-May-2018
                   3689: 
1.2       rubashki 3690: Редактор:
                   3691: Серафим Шибанов (Москва)
                   3692: 
1.1       rubashki 3693: Вопрос 1:
                   3694: В этом вопросе нет никакой эротики.
                   3695:    Жителей Тайваня, родившихся в 1980-х годах, которые не выдерживают
                   3696: социального давления или тяжелого труда, называют поколением ЕЕ.
                   3697: Назовите ЕЕ.
                   3698: 
                   3699: Ответ:
                   3700: Клубника.
                   3701: 
                   3702: Комментарий:
                   3703: Они легко мнутся, подобно клубнике. В этом вопросе никакой "клубнички"
                   3704: нет.
                   3705: 
                   3706: Источник:
                   3707: https://ru.wikipedia.org/wiki/Клубничное_поколение
                   3708: 
                   3709: Автор:
                   3710: Серафим Шибанов (Москва)
                   3711: 
                   3712: Вопрос 2:
                   3713: Согласно тексту песни современной панк-группы, на улицах пустынно, все
                   3714: взрослые убиты, а кругом раскидана ОНА. Другая ОНА - персонаж Джанни
                   3715: Родари. Назовите ЕЕ.
                   3716: 
                   3717: Ответ:
                   3718: Кукуруза.
                   3719: 
                   3720: Комментарий:
                   3721: Песня группы "Электрозомби" под названием "На кукурузном поле" содержит
                   3722: аллюзии на рассказ "Дети кукурузы" Стивена Кинга. Тетушка Кукуруза -
                   3723: персонаж сказки "Джельсомино в Стране лжецов" Джанни Родари, в другой
                   3724: сказке которого еще больше персонажей с "овощными" именами.
                   3725: 
                   3726: Источник:
                   3727:    1. Электрозомби - На кукурузном поле.mp3.
                   3728:    2. Дж. Родари. Джельсомино в Стране лжецов.
                   3729: http://flibusta.is/b/332952/read
                   3730: 
                   3731: Автор:
                   3732: Даниил Шункевич (Минск)
                   3733: 
                   3734: Вопрос 3:
                   3735: Амансио и Розалия очень любили фильм "Грек Зорба" и даже назвали свой
                   3736: магазин в честь него, однако в итоге из-за проблем с регистрацией
                   3737: пришлось найти новое название. Напишите это новое название.
                   3738: 
                   3739: Ответ:
                   3740: "Zara".
                   3741: 
                   3742: Зачет:
                   3743: "Зара".
                   3744: 
                   3745: Комментарий:
                   3746: Новое название недалеко ушло от старого в части набора букв. Модный
                   3747: бренд "Zara" [зАра] создан испанцами Амансио Ортегой и Розалией Мера.
                   3748: 
                   3749: Источник:
                   3750: http://www.woman.ru/fashion/clothes/article/58633/
                   3751: 
                   3752: Автор:
                   3753: Серафим Шибанов (Москва)
                   3754: 
                   3755: Вопрос 4:
                   3756: Персонаж одного фильма, проводя параллель между своей внучкой и Марией
                   3757: Склодовской-Кюри, утверждает, что величайшие открытия совершены гениями,
                   3758: встречающимися на планете... Закончите его реплику двумя словами,
                   3759: начинающимися на одну букву.
                   3760: 
                   3761: Ответ:
                   3762: "... реже радия".
                   3763: 
                   3764: Комментарий:
                   3765: Мария Склодовская-Кюри известна открытием радия.
                   3766: 
                   3767: Источник:
                   3768: Х/ф "Одарённая" (2017), реж. Марк Уэбб.
                   3769: 
                   3770: Автор:
                   3771: Вадим Кузмич (Логойск)
                   3772: 
                   3773: Вопрос 5:
                   3774: Небогатый герой одного романа сумел спастись с тонущего "Титаника".
                   3775: Впоследствии ему часто приходилось делать то же, что и министру.
                   3776: Назовите этого министра.
                   3777: 
                   3778: Ответ:
                   3779: [Александр Федорович] Керенский.
                   3780: 
                   3781: Комментарий:
                   3782: В спасательные шлюпки на "Титанике" в первую очередь попадали дамы и
                   3783: пассажиры, ехавшие первым классом. По идее, небогатый герой должен был
                   3784: утонуть. Друзья подозревали, что ради попадания в шлюпку он переоделся в
                   3785: женское платье, а он всю жизнь доказывал, что это не так. Тем же часто
                   3786: занимался министр Временного правительства Александр Керенский.
                   3787: 
                   3788: Источник:
                   3789: Дж. Барнс. История мира в 10 1/2 главах.
                   3790: http://flibusta.is/b/120145/read
                   3791: 
                   3792: Автор:
                   3793: Евгений Лешкович (Минск)
                   3794: 
                   3795: Вопрос 6:
                   3796: В 1872 году Гюстав Флобер писал, что после ИХ ухода дом пришлось
                   3797: дезинфицировать. Судя по дате, имелись в виду люди. Назовите ИХ.
                   3798: 
                   3799: Ответ:
                   3800: Пруссаки.
                   3801: 
                   3802: Комментарий:
                   3803: Флобер, конечно, имел в виду прусских солдат. В русском языке пруссаками
                   3804: называют еще и тараканов.
                   3805: 
                   3806: Источник:
                   3807: Дж. Барнс. Попугай Флобера. http://flibusta.is/b/321828/read
                   3808: 
                   3809: Автор:
                   3810: Евгений Лешкович (Минск)
                   3811: 
                   3812: Вопрос 7:
                   3813: На фреске работы Джотто персонаж размахивает ногами, а верхней половины
                   3814: тела не видно. Кто этот персонаж?
                   3815: 
                   3816: Ответ:
                   3817: Иона.
                   3818: 
                   3819: Комментарий:
                   3820: Фреска называется "Иона". Верхняя половина тела уже находится в пасти
                   3821: кита. Подавляющее большинство фресок посвящено библейским сюжетам.
                   3822: 
                   3823: Источник:
                   3824: https://arabena.livejournal.com/283780.html
                   3825: 
                   3826: Автор:
                   3827: Евгений Лешкович (Минск)
                   3828: 
                   3829: Вопрос 8:
                   3830: Герои современной сказки после неудачной охоты на НЕГО вымещают злобу на
                   3831: девушке, которая служила им приманкой. Таким образом они окончательно
                   3832: лишают себя надежды на удачную охоту, несмотря на то что девушка
                   3833: остается жива. Назовите ЕГО одним словом.
                   3834: 
                   3835: Ответ:
                   3836: Единорог.
                   3837: 
                   3838: Комментарий:
                   3839: Обозленные охотники насилуют девушку, которая до этого была невинной и
                   3840: служила им приманкой на единорога, укротить которого, как известно,
                   3841: может только девственница.
                   3842: 
                   3843: Источник:
                   3844: П. Бормор. Книга на третье. http://flibusta.is/b/191906/read
                   3845: 
                   3846: Автор:
                   3847: Даниил Шункевич (Минск)
                   3848: 
                   3849: Вопрос 9:
                   3850: Джон Уилкс Бут ненавидел Линкольна и считал его тираном, угрожающим
                   3851: существованию республики. В своем дневнике Бут называет день покушения
                   3852: на Линкольна римским словом. Каким именно словом?
                   3853: 
                   3854: Ответ:
                   3855: Иды.
                   3856: 
                   3857: Комментарий:
                   3858: Бут сравнивал Линкольна с Цезарем, а себя - с Брутом. Цезарь был убит в
                   3859: мартовские иды 44 года до н.э.
                   3860: 
                   3861: Источник:
                   3862: М. Бирд. SPQR: История Древнего Рима.
                   3863: https://books.google.ru/books?id=kGeYDQAAQBAJ&pg=PT288#v=onepage&q&f=false
                   3864: 
                   3865: Автор:
                   3866: Евгений Лешкович (Минск)
                   3867: 

FreeBSD-CVSweb <freebsd-cvsweb@FreeBSD.org>